SlideShare una empresa de Scribd logo
1 de 18
Medicine
                    National Test                                    1. ชายไทยอายุ 25 ป มีประวัติชักทั้งตัวมา 2 ครั้ง on phenytoin 300 mg/day หลังจากนั้น
                                                                     ไมมีอาการชักอีก , 3 วันมีไข ชักทั้งตัว , CT : calcified 3 mm. , no edema รอบๆ ถาม
                        2010                                         Management



                                                                                                      6
                                                                                A. albendazole                          B. ใหยาเดิมตอ




                                                                                                   11
                                                                                C. เปลี่ยนเปน Na Valproate             D. เปลี่ยนยาเปน ...



                                                                              I
                                                                                E. surgery เอา calcification ออก



                                                                            S
                                                                     2. ผูปวยอายุ 60 ป สูบบุหรี่ วันละ 20 มวนมานานหลายป มีหอบเหนื่อย ไข เสมหะมาก

                         BY SI116
                                                                          t
                                                                     มา 1 วัน ผล CXR : พบเปน Perihilar mass (นาจะเปน CA lung) ถามวารักษาอยางไรตอ




                                                            igh
                                                                                A. chemotherapy                         B. radiotherapy




                                                         yr
                                                                                C. surgery                              D. O2 therapy
                         ฉบับราง                                    3. ผูปวยไอมีเสมหะ 2-3 ป กินยาเทาไรก็ไมหาย ปากเหม็นมาก



                                        p
                                                                     PE : coarse crepitation BLL ถาม diagnosis



                                      o
                                                                                A. chronic bronchitis                   B. COPD



                          C
         หมายเหตุ ใชสําหรับสอบNational test วันอาทิตยที่10/10/53              C. Bronchiectasis                       D. lung abscess
      ขอบคุณเพื่อนๆSI116ทุกคนที่ชวยกันจําขอสอบ                     4. หญิง 75 ป เปน Hypertension รักษาดวยยา Hctz(25) 1x1 คุม BP ดีมาตลอด วั้นนี้ลุกจาก
                                                                     ที่นอนไมขึ้น ญาติพามารพ. BP 130/80 , P 90 /min , alert , on wheel chair , proximal m.
     ทําโดย กอง(010),First(019),นันทวุฒิ(089),การ(104),             weakness , DTR & sensory intact ถามวาเกิดจากอะไร
บอส(139),Bank(มง)(144),โย(191),Golf(212)                                       A. hyponatremia                         B. hypokalemia
      ยังไงก็ขอใหสอบผานกันทุกคนนะ                                             C. hypomagnesemia                       D. hypercalcemia
                                                                                E. rhabdomyolysis
5. ชาย 55 ป เปน hepatitis C cirrhosis มีไข ตัว/ตาเหลืองมา 3 วัน ทองโตมา 3 wks.              A. tetrodotoxin                      B. Botulinum toxin
PE : T 38 C , Jaundice , tense ascites                                                          C. Cyanide                           D. Cabamate
Ascitic fluid study : WBC 2,200 , PMN 60% , mononuclear cell 40% , gram stain neg,              E. Organophosphate
ascites fluid/serum albumin = 0.9/2.5 mg/dl จง management                            9. peripheral blood smear พบ plasmodium falciparum(multiple ring) (รูป) ถามยาที่ใชใน




                                                                                                                      6
           A. somatostatin + albumin 50 g/day                                        การรักษา



                                                                                                                    1
           B. metronidazole 500 mg tid                                                          A. Artesunate + mefloquine           B. Quinine




                                                                                              I                    1
           C. cef-3 100 mg+ somatostatin IV drip                                                C. Chloroquine + Primaquine          D. Chloroquine




                                                                                            S
           D. ciprofloxacin 500 mg + telipresin IV q 6 hr                                       E. Mefloquine




                                                                                          t
           E. cef-3 2 g IV OD                                                        10. ผูปวยสูงอายุ มี U/D เปน HT, glaucoma กินยาไมทราบชนิด แลวมารพ.ดวยอาการซึม



                                                                 h
6. ชายอายุ 55 ป สูบบุหรี่มาก barrel-shape chest , prolong expiratory phase จะสง    ลง PE : alteration of consciousness




                                                               ig
investigation อะไร                                                                   Lab : Na 108, K 3.0, Cl 80, HCO3 35 คิดวาอาการเหลานี้นาจะเกิดจากยาตัวใด




                                                            yr
           A. pulmonary function test B. CXR                                                    A. Acetazolamide                     B. Spironolactone




                                            p
           C. CT chest                                                                          C. HCTZ                              D. Hydralazine




                                          o
7. ชายอายุ 20 ป พบนอนหมดสติ(จากไฟไหม)                                                         E. Triamzinorone




                              C
V/S : P 68 /min , BP 90/70 mmHg , RR 12 /min , no response to deep pain , พบ black   11. ผูปวยชายสูงอายุ เปน ESRD on hemodialysis สัปดาหละ 2 ครั้ง มีอาการใจสั่น หอบ
soot at mouth&thumb ถามวาจะตรวจอะไรเพื่อการวินิจฉัย                                 เหนื่อย ตรวจรางกายพบวามี BP สูง, puffy eyelid, pitting edema both legs
           A. O2 sat                               B. Venous blood gas               Lab : BUN 60, Cr 6, Na 136, K 6.8, Cl 110, HCO3 27 ควรจะทําอะไรเปนอันดับแรก
           C. Electrolyte                                                                       A. NaHCO3                            B. Hemodialysis
8. หญิง 25 ปมีอาการชารอบปาก หายใจไมสะดวก , อึดอัด หลังจากรับประทานกวยเตี๋ยว                  C. Calcium gluconate                 D. Kayexalate
ลูกชิ้นปลามา 1 ชม. หลังจากนอนสังเกตอาการหายใจลําบากมากขึ้นเรื่อยๆ จนหยุดหายใจ                   E. Glucose + insulin
ผูปวยไดรับสารพิษอะไร
12. ผูปวยหญิง มีผื่นที่ศีรษะลักษณะปนสีน้ําตาลที่ศีรษะ. รูปวงกลม ขนาด 5 x 3 cm ตรง     16. ผูชายอายุ 60 ป admit ICU ดวย pneumococaal pneumonia with respiratory failure
กลางเห็นเปน capillary ไมมีผมขึ้น, มี scale ตรวจรางกายอื่นๆปกติ วินิจฉัยวาอะไร         อาการดีขึ้น ตอมาสําลักอาหารมีไขสูง ไอ หอบเหนื่อยมากขึ้น T 36 CXR มี infiltration,
           A. Taenia capitis                        B. Discoid lupus erythematosus        sputum C/S : gram neg rods with bipolar staining ให ATB อะไร
           C. psoriasis                             D. Kerion                                        A. Amikacin                          B. Augmentin




                                                                                                                           6
           E. Scleroderma                                                                            C. Ciprofloxacin                     D. Cef-3



                                                                                                                         1
13. ชายอายุ 50 ป poor controlled HT วันนี้ปวดหัว ออนเพลีย BP 200/120 ตรวจรางกาย มี                E. Imipenam




                                                                                                   I                    1
sign of congestive heart failure, s3 gallop, retinal hemorrhage จงmanage เบื้องตน        17. ผูปวยชายออนแรงซีกขวา เปนมากขึ้นเรื่อยๆมา 7 วัน ตรวจพบวา no stiff neck,




                                                                                                 S
           A. propanolol                            B. nitroprosside IV                   positive white patch tongue & oral mucosa จงใหการรักษา




                                                                                               t
           C. hydrolazine IV                        D. Morphine                                      A. Amphotericin B                    B. Clotrimazole



                                                                     h
           E. digoxin                                                                                C. Co-amoxiclav                      D. TMP + SMZ




                                                                   ig
14. ผูปวยเคยไดรับการ diag วาเปน Rheumatic fever ตองใหpenicillin จนถึงเมื่อไร                  E. Ceftriaxone




                                                                yr
           A. ใหจนอายุ 50                          B. ใหเวลาทําฟน                      18. ผูปวยชาย จ.หนองคาย มีอาการปวดบวมแดงรอน ที่บริเวณขาขางซาย drain ไดหนอง




                                              p
           C. ใหตลอดชีวิต                                                                พบ bacteria เปน Gram negative slender rod ควรใหยาอะไร




                                            o
15. หญิงอายุ 18 ป มีไข, ปวดขอเทาวาย และขอมือขวามา 4 วัน                                        A. Cefazidime                        B. Gentamicin




                                C
PE : T 40 c, BP 110/70 mmHg, PR 80/min, RR 20/min, tenderness and swelling at Left                   C. Amikacin                          D. Ceftriaxone
ankle joint and right writst joint                                                        19. ผูปวยชาย ชอบกินเหลาประจํา ตรวจพบมี right upper lobe pneumonia, Gram stain
ทํา arthrocentesis พบ turbid synovial fluid 5 ml, WBC 25000 (N 90%), Gram stain no        เปน numerus Gram negative & positive ควรใหการรักษาอยางไร
organism seen จงใหการรักษา                                                                          A. Co-amoxiclav                      B. Levofloxacin
           A. NSAIDs oral                           B. Ceftriaxone V                                 C. Clarithromycin                    D. Ceftriaxone
           C. Penicillin G                          D. Intraarticular steroid injection   20. ผูชาย 25 ป บริจาคเลือดตรวจพบ HBsAg และ Anti HBC neg, LFT ปกติ ทําอยางไร
                                                                                                     A. ตรวจ HBsAg ซ้ําใน 6 month         B. ให lamivudine
C. ตรวจ viral load                    D. ให hep B immunoglobulin             25. Umbilical vesicle with crust ควรใหยาอะไร
           E. ให HBV vaccine                                                                       A. Acyclovir
21. ผูหญิงเหนื่อยงายมา 4 เดือน ตรวจรางกายพบ moderately pale, koilonikis, CBC Hct                 B. Cloxacillin
22%, wbc 3,500 N 65 L 35 Plt 80,000, PBS ดังรูป (มี Hypersegmental neutrophil, Plt ไม   26. ผูปวยหญิงมีอาการเมาเรือ ทานจะใหยาใด




                                                                                                                           6
มี) จงใหการวินิจฉัย                                                                                A. Dimenhydrinate                                B. Ceterizine



                                                                                                                         1
           A. Apastic Anemia                     B.Myelopathisis anemia                  27. ผูปวยหญิงมีผื่นกลมขึ้นทั่วตัว ผมรวงมีขุย บริเวณรอบๆเห็นเปนเสนเลือด




                                                                                                  I                     1
           C. MDS                                D. iron def                                        A. Tinea capitis                                 B. Discoid rash




                                                                                                S
           E. megaloblastic anemia                                                       28. ผูปวยมีประวัติไอปนเลือดเรื้อรัง ตรวจรางกายพบ dilated superficial vein ที่หนาอก




                                                                                              t
22. ผูชายอายุ 22 ป asthma พน Steroid, Salbutamol, ปกติ expire peak 400, RR 35 มี      ขางขวา (รูป) ใหการวินิจฉัย



                                                                     h
asthma attack -> Peak 220 พน salbutamol x II แลวได PEF 320 RR 25 จะทํายังงัยตอ                  A. SVC syndrome




                                                                   ig
           A. Admit                              B. เพิ่มยา steroid                      29. ผูปวยอายุ 50 ป เปน DM ได Metformin + …, BMI 34, FBS 130, HbA1C 8.3 จงให




                                                                yr
           C. เพิ่มยา salbutamol                 D. กลับบาน                             การ management




                                              p
23. ผูปวยหลังรับประทานอาหารทะเลแลว BP drop, มี mucosa บวม, lung clear, มีผื่นขึ้น                A. เพิ่มขนาดยา metformin                         B. เพิ่มยา glipizide




                                            o
ตามตัว ควรใหการรักษาอยางไร                                                                        C. เพิ่มยา Acarbose




                                C
           A. IV corticosteroid                  B. IM adrenaline                        30. ผูปวยหญิงอายุ 30 ป moderately pale, mild icteric sclera
           C. IV CPM                             D. IV hydrocortisone                    Hct 25, WBC 5000, Plt 200000, RBC : anisocytosis & Poikilocytosis 2+ with schitocyte
24. ผูปวยเดินทางไปเที่ยวที่วนอุทยาน 5 wk หลังเดินทางกลับได 2 วันมีไขหนาวสั่น ควร     2+ ควรทําการตรวจอะไรเพิ่มเติม
ทํา investigation ใดเพิ่มเติม                                                                       A. Hb typing                                     B. G-6-PD deficiency
           A. CBC, H/C                                                                              C. BMA                                           D. Coomb’s test
           B. Peripheral blood smear for malaria                                                    E. osmotic …
           C. Blood for rickettsial study
31. ชาย 16 ป ปวดเขา ตรวจ Rt. Knee มี swelling & warm, ผล PT ปกติ & prolonged PTT              A. vitamin D excess                            B. primary hyperparathyroid
ใหการรักษาเบื้องตนอยางไร                                                                     C. secondary hyperparathyroid                  D. tertiary hyperparathyroid
           A. Cryoprecipitate                               B. FFP                   37. ผูปวยหญิง 60 ป เปน DM, HT ไดรับประทานยา enalapril, ASA, HCTZ มานาน 1 ป
           C. Cryoprecipitate + platelet                                             6 เดือนกอนมีปวดศีรษะ ตรวจพบ uric acid ในเลือดสูง ไดรับยา allopurinol และ




                                                                                                                       6
32. ผูปวยมีอาการถายอุจจาระบอย ตรวจ AFB พบเชื้อดังรูป (Cryptosporidium) ใหการ    paracetamol มารับประทาน



                                                                                                                     1
วินิจฉัย                                                                             1 สัปดาหกอน มี MP rash ขึ้น ถามวานาจะเกิดจากยาใดมากที่สุด




                                                                                              I                     1
           A. Cryptosporidium                               B. Isospora belli                   A. ASA                               B. Enalapril




                                                                                            S
           C. Stongyloides stercoralis                                                          C. HCTZ                              D. allopurinol




                                                                                          t
34. ผูปวยหญิงมาดวยเรื่องไข เจ็บคอ                                                           E. paracetamol



                                                                 h
PE : no thyroid enlargement, tenderness at thyroid gland                             38. ผูปวยมีอาการ migraine ถามวายาใดที่ใชปองกันอาการ migraine ไดดีที่สุด




                                                               ig
TFT : FT4 & T3 ↑, TSH ↓                                                                         A. ASA                               B. Propanolol




                                                            yr
           A. Acute suppurative thyroiditis      B. Subacute thyroiditis                        C. Indolmethacin                     D. Etgotamine




                                            p
           C. Grave’s disease                    D. Hashimoto’s thyroiditis          39. ผูชายอายุ 25 ป ถายเหลวหลังรับประทานอาหารมา 6 เดือน ไมมีอาการอื่นๆ นน. ไม




                                          o
           E. toxic goiter                                                           ลด ตรวจอะไร




                              C
35. ชาย 20 ป dx. Grave disease รักษาดวยยา PTU 2x3 หลังรักษามา 3 mo. มา F/U FT4              A. Stool exam                         B. Colonoscopy
สูง , TSH ต่ํา , WBC ~ 500 จง management
                                                                                              C. Thyroid Function test              D. GI study
           A. ใหยาขนาดเทาเดิม                  B. ลดขนาดยาลงเปน 1x3
           C. เพิ่มยาเปน 3x3                    D. เปลี่ยนยาเปน metrimazole        40. ผูปวยชายอายุ 18 ป หมดสติในงานรับนอง มีกลิ่น alcohol, HR60/min, BP 100/60
           E. เปลี่ยนการรักษาเปน Radioactive iodine                                 mmHg ควรทําอยางไรอันดับแรก
36. ผูปวยชายอายุ 70 ป กินวิตามินเสริมประจํา มีประวัตินิ่วเมื่อ 10 ปกอน ตรวจพบ              A. ตรวจ ketone                     B. ตรวจ electrolyte
calcium 11, PTH ↑ ถามวาเกิดจากอะไร                                                             C. 50% glucose IV push             D. CT brain
41. ผูปวยชายอายุ 75 ป BP 140/80 mmHg, PR 38 /min, EKG เปน bradycardia ~ AV          46. ทองเสียเรื้อรัง เดิมเคยทองผูก (คนแก) ทองมี saucesage-like mass RLQ , PR มีอึแข็ง
block ควรทําอยางไรตอไป                                                                +อึเหลวปนกัน ถาม management (v/s stable , ไมมีอาการอื่นๆ)
           A. Atropine                              B. Cardioversion                               A. ORS                                  B. IV fluid
           C. Defibrillation                        D. Adenosine                                   C. loperamide                           D. NSS enema




                                                                                                                          6
42. ผูปวยชายอายุ 70 ป มีมือสั่นเวลาหยิบชอน ตรวจรางกายมี normal gait จงใหการ       47. ผูปวยชาย ทําอาชีพขับรถสงของ มีอาการคันหู คันตา จาม น้ํามูกไหล ควรรับประทาน



                                                                                                                        1
วินิจฉัย                                                                                ยาอะไร




                                                                                                 I                     1
           A. Parkinson’s disease                   B. Essential tremor                            A. Loratadine                           B. CPM




                                                                                               S
           C. Cerebellar atrophy                    D. Normal change                    48. ผูปวยมี diarrhea เปนน้ําเหลวมา 2 วัน มี BP ทานั่ง 80/60 mmHg, ทานอน 120/70




                                                                                             t
43. ผูปวยชายอายุ 40 ป มีซีด ออนเพลียมากขึ้นมา ??? เดือน                             mmHg, poor skin turgor, JVP = 0, urine Sp.gr. 1.030, urine Na 10 mEq/L ใหการวินิจฉัย



                                                                   h
PE : pale, jaundice                                                                                A. Pre-renal failure                    B. Acute interstitial necrosis




                                                                 ig
CBC : Hct ↓ , WBC ↓ , Plt ↓                                                                        C. Acute glomerulonephritis             D. Acute tubular necrosis




                                                              yr
UA : Sp.gr. 1.015, RBC 5-10 cells/HF                                                    49. ผูปวยชาย เปน DM ตรวจพบมีปนดําๆที่จมูกและปาก ทํา KOH พบ non-septate




                                             p
จงใหการวินิจฉัย                                                                        hyphae with branching




                                           o
           A. G6PD deficiency                       B. Aplastic anemia                             A. Clotrimazole                                    B. Itraconazole




                               C
           C. AIHA                                  D. PNH                                         C. Amphotericin B                                  D. Ketoconazole
44. ผูปวยกลืนน้ํายาลางหองน้ําที่เปนดาง ควรใหการ management อยางไร               50. ผูปวย car accident ตอง exploratory laparotomy + splenectomy ระหวางผาตัดได PRC
           A. Endoscopy                             B. Observe                          1 U, vital sign stable สงขึ้นไป admit บนตึกผูปวย ไดให PRC อีก 1 U ปรากฏวาผูปวยมี
           C. ประเมิน score                                                             ไขหนาวสั่น CVA tenderness, BP ↓, Pulse rate ↑, RR 24 /min ควรทําอยางไร
45. ผูปวยชายอายุประมาณ 55 ป มี DLP ยาใดตอไปนี้ที่มี cost benefit ในการปองกัน CAD              A. ให NSS IV                                      B. ให hydrocortisone
           A. Simvastatin                           B. Cholestyramine                              C. ให dexamethasone                               D. หยุดการใหเลือด
                                                                                                   E. ให adenosine
51. ผูปวยชาย มาดวยเรื่อง HT BP 160/100 mmHg ออนแรง ตรวจพบมี Na 152, K 2.2 จง                                       Pedriatric
ใหการวินิจฉัย
                                                                                   02 CNS
           A. primary hyperaldosteronism     B. pheochromocytoma
                                                                                   1. ผูปวยเด็กอายุ 10 ป มีอาการเหมอ ครั้งละ 10-20 วินาที วันละประมาณ 10 ครั้งใหการ
           C. Cushing syndrome
                                                                                   management



                                                                                                                     6
                                                                                               A. Valproic acid                      B. Diazepam




                                                                                                                  11
                                                                                               C. Phanobarbital                      D. Phenytoin



                                                                                            I
                                                                                   02 RS



                                                                                          S
                                                                                   2. เด็กอายุ 10 เดือนมาดวยอาการหอบเหนื่อย ฟง lungs ได bilateral wheezing พน



                                                                                        t
                                                                                   salbutamol ไป 1 dose อาการไมดีขึ้น DX




                                                              igh
                                                                                               A. croup                              B. bronchitis




                                                           yr
                                                                                               C. bronchiolitis                      D. viral pneumonia
                                                                                               E. severe asthmatic attack



                                           p
                                                                                   3. ผูปวยเด็กชาย อายุ 10 ป มีอาการไขหวัดมา 3 เดือน ไมมีอาการปวดศีรษะ มีอาการไอ



                                         o
                                                                                   น้ํามูกไหล



                              C
                                                                                   vital sign stable
                                                                                   HEENT : no posterior nasal drip, pale nasal mucosa with mucoid discharge จะสงตรวจ
                                                                                   อะไรตอไป
                                                                                               A. CBC                                B. Film paranasal sinus
                                                                                               C. Allergy test                       D. nasal discharge culture
4. ผูปวยเด็กเปน asthma on Budesonide 800 mg/day, Salbutamol ตอมามีอาการหอบ               8. ped เด็กอายุ 5 ป เปน dengue hemorrhagic fever BP 90/70 P 120 BW 20 kg ใหสารน้ํา
เหนื่อย (Acute asthmatic attack) Peak flow 200 (เดิมได 400) ได oxygen + Salbutamol         ใด
NB ตอมา Peak flow เพิ่มขึ้นเปน 320 จําทําอยางไรตอไป                                                 A. 5% D/N/2 rate 100 ml/hr
            A. ใหกลับบาน                          B. เพิ่ม dose Budesonide เปน 1200                  B. 5% D/N/2 rate 200 ml/hr




                                                                                                                              6
            C. เพิ่ม dose Sulbutamol                D. ให oral prednisolone x 5 days                   C. 5% D/NSS rate 100 ml/hr



                                                                                                                            1
5. หญิง 15 ป asthma 5 ป มีไขหวัดมา 1 วัน หอบเหนื่อย RR 30 /min , P 110 /min , BP                     D. 5% D/NSS rate 200 ml/hr




                                                                                                      I                    1
140/90 mmHg , ได salbutamol nebule จะไดอะไรตอ                                                        E. 5% D/N/3 rate 100 ml/hr




                                                                                                    S
            A. IV theophylline                      B. subcut. Terbutaline                   9. เด็กทองเสียเปนน้ํา 10 ครั้ง/วันมา 2 วัน อําเจียน 2 ครั้ง




                                                                                                  t
            C. IV dexa                              D. beclomethasone nebule                  BP 90/50 mmHg , CBC N80% , L 20% เกิดจากเชื้ออะไร



                                                                        h
5. ผูปวยเด็กอายุ 10 เดือน มีหอบ ไข ไอ, wheezing both lung พน salbutamol แลวอาการ                   A. Salmonella                            B. Vibrio chorela




                                                                      ig
ไมดีขึ้น จงใหการวินิจฉัย                                                                              C. Strongeloids                          D. Isospora




                                                                   yr
            A. croup                                B. Bronchiolitis                                    E. Amoebesis




                                                p
            C. pneumonia                                                                     02 Endocrine




                                              o
6. เด็กอายุ 1 ป มีไขต่ําๆ ไอเสียงกอง มีน้ํามูกไหล                                         10. เด็กหญิงอายุ 10 ป ไมมีอาการอะไรนอกจาก nodular goiter 2cm นอกจาก TFT แลวจะ




                                 C
PE : inspiratory stridor, subcostal & intercostals retraction จงใหการ management            ทําอะไร
            A. Adrenaline IV                                   B. Salbutamol NB                         A. urine iodine                          B. FNA
            C. Dexamethazone                                   D. ET tube                               C. thyroid scan
02Infectious                                                                                 11. เด็กอายุ ... ป , ชวงนี้กินมาก อวนมากขึ้น ตรวจพบ abdominal striae ถามวาจะเกิด
7. เด็ก 6 ป ไขสูง 3 วัน ปวดเมื่อยตัว ถายเหลวเล็กนอย ครูประจําชั้นมา รพ ที่ รร มีเด็กมี   ภาวะแทรกซอนใด
อาการเหมือนกัน 3-4 คน จงใหการวินิจฉัย                                                                  A. hypoglycemia                          B. hypokalemia
            A. Influenza                            B. DHF                                              C. hypertension
12. เด็กอายุ 14 ปเปนเบาหวาน อาเจียนมาก BP 90/70, P 110/min, PRR 25/min มา รพ ให        16. เด็กอายุ 10 ป เปน Molluscum contaciosum ที่หลัง ขนาด 0.3-0.8 cm ควรใหการรักษา
IV rate                                                                                   อยางไร
           A. 0.9% NSS 600 cc/hr                                                                     A. extraction และบีบออก              B. ทา sodium thiosulphate
           B. 0.9% NSS 300 cc/hr                                                                     C. benzyl peroxide                   D. Tzank’s smear ผูปวยเด็กมีผื่นตุมแดง




                                                                                                                            6
           C. 0.45% NSS + NaHCO3 600 cc/hr                                                17. กดเจ็บที่บริเวณขาทั้งสองขาง ใหการวินิจฉัย



                                                                                                                          1
           D. 0.45% NSS + NaHCO3 300 cc/hr                                                           A. Erythyma nodosum




                                                                                                   I                     1
           E. RLS 300 cc/hr                                                               02Nutrition




                                                                                                 S
13. เด็กอายุ 5 ป ทองอืดและผูกบอย ผูปวยมีลักษณะลิ้นจุกปาก (รูป) ตรวจวามี BW = P50,   18. ผูปวยเด็กหญิง term กินนมมารดาอยางเดียว อายุขณะนี้ 12 wk มาตรวจรางกายพบ Hct




                                                                                               t
Ht < P3 จงใหการวินิจฉัยเบื้องตน                                                         32%, WBC&Plt normal, reticulocyte count 2% จะทําอะไรตอไป



                                                                     h
           A. Achondroplasia                     B. Thalassemia                                      A. Fe supplement                               B. Folate supplement




                                                                   ig
           C. Glycogen storage disease           D. Hypothyroidism                                   C. Vitamin E                                   D. Reassure




                                                                yr
           E. Hypoparathyroidism                                                          02Neonate




                                              p
02 SKIN                                                                                   19. เด็กแรกเกิดน้ําหนักนอย birth weight 1400 g, jaundice at day2, Apgar score 6,7 ได on




                                            o
14. ผูปวยเด็กกอนนอนอาการปกติดี หลังตื่นมามีตุมแดงที่ขาและลําตัว จงใหการวินิจฉัย      phototherapy อะไรเปนปจจัยที่ทําใหตองตรวจหูในอนาคต




                                C
           A. contact dermatitis                           B. Herpes zoster                          A. low birth wight                             B. jaundice
15. dermato เด็กชาย 10 ป มีตุมสีขาวมา 2 เดือน                                                      C. on Phototherapy                             D. Apgar score ต่ํา
PE : multiple discrete umbillicated papule ผื่นเปนมากขึ้นตามรอยเกา                       20. เด็กแรกคลอดไมหายใจ ตัวเขียว ชวยหายใจ บีบ bag นาน 5 นาที เขียวมากขึ้น หนาอก
           A. Extraction                         B. Topical Na thiosulfate                โปง ทองแฟบ HR 60/min เสียงหัวใจฟงชัดดานขวา ควรทําอยางไร
           C. Topical Benzyl benzoate            D. Itraconazole                                     A. Adrenaline IV                     B. เจาะ Lt. pleura
           E. Acyclovir                                                                              B. ET tube
21. ผูปวยเด็กแรกเกิดหนัก 4200 g มารดาเปน DM พบวามีใบหนาแดงคล้ํา Hct = 70% ,                                           OB-GYN
blood sugar = 45 ควรทําอะไรตอไป
                                                                                      03GYN
           A. Partial blood transfusion                      B. Blood testing
                                                                                      1. ผูปวยหญิง G1 ขาดประจําเดือนมา 2 เดือน มีเลือดออกกระปริดกระปรอยทางชองคลอด
           C. NSS iv                                         D. 10% D/W
                                                                                      ปวดทองนอย ตรวจรางกายมี vital sign stable, BP 80/50 mmHg



                                                                                                                        6
           E. early feeding
                                                                                      PV : adnexal mass, bulging cul-de-sac, tender at cervical excitation



                                                                                                                      1
22. เด็กหลังคลอด 10 hr พบมี Jaundice MB = 10 , Hct = 48%



                                                                                                                     1
                                                                                      ใหการวินิจฉัย



                                                                                               I
           a. on photo                   b. exchange
                                                                                                 A. Rupture ectopic pregnancy



                                                                                             S
02Emergency
                                                                                      2. หญิง GA 30 wks , bp สูง , ลูกไมมี FHR , CBC : MAHA , จะตองเจาะอะไร



                                                                                           t
23. ผูปวยเด็กขณะเลนลูกแกว มีไอ สําลัก แตยังสามารถเลนไดตามปกติ ตรวจมี SpO2
                                                                                                 A. LFT                                  B. Lupus andicoag



                                                                   h
94% ควรทําอยางไร




                                                                 ig
                                                                                                 C. D-dimer
           A. ใส ET tube                          B. tracheostomy




                                                              yr
                                                                                      3. ใหรูป candida albicans มีประวัติตกขาว คัน ใหการรักษา
           C. direct laryngoscopy + bronchoscopy
                                                                                                 A. amoxicillin                          B. metronidazole



                                             p
02 Delvolopement
                                                                                                 C. Norfloxacin                          D. vaginal tablet metronidazole



                                           o
24. ผูปวยเด็ก มีลักษณะซุกซนมาก อยูไมนิ่ง เรียกไมหัน ชอบเลนคนเดียว ควรสงตรวจ
                                                                                                 E. vaginal tablet clotrimazole



                               C
อะไร
                                                                                      4. ผูปวยหญิงอายุ 30 ป ยังไมเคยตั้งครรภ ไมมีอาการใดผิดปกติ มาตรวจรางกาย ทํา
           A. Denver test
                                                                                      ultrasound พบวามี uterus size = 11 cm, มี frbroid mass 3x2 cm ที่ anterior wall และ 2x2
02KUB
                                                                                      cm ที่ posterior wall ควรทําอยางไร
25. เด็กชายอายุ 1 ป มีไขมา 1 วัน ไดวินิจฉัยเปน UTI ปสสาวะพบ numerous WBC,
                                                                                                 A. F/U 3 months                         B. GnRH agonist
Culture E. coli ให antibiotic นาน 2 สัปดาห อาการดีขึ้น ควรทําการสืบคนใดเพิ่มเติม
                                                                                                 C. Uterine artery embolization          D. Myomectomy
           A. VCUG                                           B. IVP
                                                                                                 E. Hysterectomy
           C. urine culture q 1 mth
5. ผูปวยหญิง ตรวจรางกายพบมีลักษณะบวมแดงที่ vulva สงสัย masupial cyst ควรทํา                      E. induction of labor
อะไรตอ (รูป)                                                                            9. ผูปวยหญิงตั้งครรภ เคยไดรับวินิจฉัยเปน placenta previa เมื่อ GA 32 wk ไดรับฉีด
           A. Reassure                           B. cruciate incision & drainage         steroid IM ไป เมื่อ GA 34 wk ครั้งนี้มีเลือดออกชุมผาอนามัย 2 ผืน เมื่อ 4 hr กอนมารพ.
           C. Enucleation                        D. Excision                             แตไมมีอาการเจ็บครรภ ไมมีมดลูกบีบตัว บานอยูหางจากรพ.เดินทางประมาณ 1 hr ผูปวย




                                                                                                                           6
           E. Vulvectomy                                                                 ตองการคลอดตามกําหนดอีก 1 สัปดาห ควรทําอยางไรตอไป



                                                                                                                         1
6. ผูปวยหญิง มีปญหามีบุตรยากมา 4 ป มีประวัติถูก rape มา 5 วันกอนและตอมาตั้งครรภ              A. Admit + observe




                                                                                                  I                     1
ควรทําอยางไร                                                                                       B. Discharge แลวนัดมาผาตัดคลอดตามนัด




                                                                                                S
           A. abortion                           B. ใหผูปวยปรึกษาสามี                            C. ผาตัดคลอดทันที




                                                                                              t
7. หญิงอายุ 17 ป ไข ปวดทอง 2 วัน                                                      10. ผูหญิง 40 ป แตงงานมา 10 ป GA 22 wks ตรวจน้ําคร่ําได 47,XY trisomy 21 ควรให



                                                                    h
PE : T39.8 c, BP 110/70 mmHg, RR 20/min, P 90/min                                        คําแนะนําแกมารดาอยางไร




                                                                  ig
Abd : rebound tenderness at lower abdomen                                                           A. ไดลูกเปนผูชาย มีโอกาสมีความผิดปกติ




                                                               yr
PV : mucopurulent discharge, cervical motion tenderness, normal uterine size, Adnexa                B. ใหขอมูลบิดาและมารดา แลวใหรวมกันตัดสินใจ




                                              p
tenderness both sides                                                                               C. สง ultrasound เพิ่มเติม




                                            o
ใหการวินิจฉัย                                                                                      D. MRI เพิ่มเติม




                                C
           A. Acute PID                          B. Acute appendicitis                              E. แนะนําใหยุติการตั้งครรภ
           C. Ruptured endometriotic cyst        D. acute enterocolitis                  11. ผูปวยหญิง GA 10wk, HBsAg positive ใหวางแผนการรักษา
03OBS                                                                                               A. avoid breast feeding                B. ให vaccine
8. ผูปวยหญิงตั้งครรภ GA 39 wk มาดวยเรื่องวาลูกไมดิ้นมา 24 hr ตรวจพบ FHR 140/min               C. ให HBIg
ควรทําอยางไร                                                                            12. ผูปวยหญิงอายุ 30 ป G1 GA 34 wk ปวดศีรษะ ตามัว BP 160/100 mmHg, urine
           A. reassure & F/U วันรุงขึ้น         B. Cesar.                               protein 3+ ตรวจ fundoscopic exam มี papilledema ควรทําอะไรเปนลําดับตอไป
           C. Oxytocin stress test               D. BPP                                             A. IM dexamethasone                    B. Induction of labor
C. Magnesium IV                                                                                            Surgery
13.. หญิง 40 ป ตั้งครรภ 5 ปวดเมนสมา 2 เดือนเศษๆ มีเลือดออกกระปบกระปอยทาง
                                                                                 1. ตอขวด ICD 3 ขวด แลวพบมีลมปุดที่ขวดที่ 2 ตลอดเวลา พอ clamp สายแรกแลวหาย
ชองคลอดมา 1 wk P.E. fundus 1/3 เหนือ pubic sym, BP 140/92 mmHg, P 102/min, HR
                                                                                 ถามวา รั่วตรงไหน
95/min ตรวจพบ bilat cystic mass at adnexa 3 cm
                                                                                 2. ผูปวยชาย 50 ป มีอาการปวดทองบริเวณลิ้นปมา 24 ชม. จากนั้นปวด RLQ กดเจ็บ



                                                                                                                  6
          A. molar pregnancy                   B. ectopic pregnancy
                                                                                 ถามวา investigation ใดแมนยําที่สุด



                                                                                                                1
          C. threaten abortion



                                                                                                               1
                                                                                            A. Plain abdomen                     B. GI follow through



                                                                                          I
                                                                                            C. Barium                            D. CT



                                                                                        S
                                                                                            E. U/S



                                                                                      t
                                                                                 3. ผูปวยหญิงอายุ 35 ป มีอาการปวดบริเวณเตานมขวา




                                                             igh
                                                                                 PE : mild swelling and tenderness LUO quadrant, no palpable mass, no axillary mass




                                                          yr
                                                                                 ควรตรวจขึ้นตนดวยอะไร
                                                                                            A. CBC                               B. CA 19-9



                                           p
                                                                                            C. mammogram                         D. FNA



                                         o
                                                                                 4. ผูปวยชาย อายุ 70 ป ตรวจพบมีตอมลูกหมากโต PR : rubbery consistency ไมมีอาการ



                             C
                                                                                 ผิดปกติใด PSA = 3 จงใหการ management
                                                                                            A. Follow up                         B. alpha-blocker
                                                                                            C. TURP                              C. 5-alpha reductase inhibitor
                                                                                 5. ผูปวยชายประสบอุบัติเหตุ on ET tube นาน 2 wk ไดรับอาหารทาง NG tube ได total
                                                                                 cal. 1800 kcal/day, TC 60 ควรไหสารอาหารแบบใด
                                                                                            A. Kcal 2000 kcal/day, TC 60
                                                                                            B. Kcal 2400 kcal/day, TC 90
C. Kcal 2800 kcal/day, TC 120                                               10. ชายอายุ 65 ป ปสสาวะกระปริดกระปรอย เบงนานกวาจะหมดมา 3 เดือน 1วันกอนมี
            D. เพิ่มไข 2 ฟอง/วัน                                                       ไข น้ํามูก กินยาเอง วันนี้ปสสาวะไมออก
            E. กินไขทุกมื้อ มื้อละ 1 ฟอง                                               PE : full bladder, prostate enlargement, rubbery consistency ควรใหการรักษาอยางไร
6. ผูปวยมีแผลที่ริมฝปากดานในขนาด 1 cm มานาน 7 วัน ตรวจพบ LN ที่คางดานซายโต                    A. ใสสายสวนปสสาวะ 3-5 วัน




                                                                                                                          6
เสนผานศูนยกลางขนาด 1 cm with tenderness ควรทําอะไรตอ                                            B. ใสสายสวนปสสาวะ 3-5 วัน + ใหยารักษา BPH



                                                                                                                        1
            A. observe                              B. LN biopsy                                    C. ใสสายสวนปสสาวะนาน 3-5 วัน + TURP




                                                                                                 I                     1
            C. incisional biopsy of oral ulcer     D. Excisional biopsy of oral ulcer               D. Suprapubic cystostomy




                                                                                               S
            E. FNA LN                                                                               E. Cystoscopy หาสาเหตุของการปสสาวะไมออก




                                                                                             t
7. ชายอายุ 50 ป, เคยปวดเอว,มีไข,ปสสาวะแสบ 2 ครั้ง/ป เคยปวดเอวราวลงขาขวาเมื่อ 2     11. ผูปวยหญิง ผาตัด thyroid mass Lt lobe เสนผาศูนยกลางขนาด 3 cm หลังผาตัดมีเสียง



                                                                    h
ปกอน 2 ครั้ง , U/A : RBC 5-10 , WBC 0-1 , BUN 15 , Cr 0.5 จะสง Investigation อะไร    แหบ กลืนลําบาก เนื่องจากอะไร




                                                                  ig
            A. urine Ca                             B. plane KUB                                    A. laryngeal edema                     B. injury to superior laryngeal n.




                                                               yr
            C. U/S KUB                              D. cystoscope                                   C. arythenoid subluxation              D. vocal cord paralysis




                                              p
            E. IVP                                                                      12. เด็กอายุ 2 ป มีกอนสีแดงที่หนังตาซาย ตอนแรกมีลกษณะคลายตุมยุงกัด ตอมากอนโต
                                                                                                                                                 ั




                                            o
8. ผูปวย trauma มีไอเปนเลือด , คลํา crepitation ไดที่คอ                             มากขึ้นเรื่อยๆลักษณะเปนกอนสีแดงคลายสตรอเบอรี่ มีหนังตาตกลงมาครึ่งหนึ่ง จะทํา




                                C
            A. ABG                                  B. intubation                       อยางไร
            C. tracheostomy                                                                         A. observe ใหกอนยุบลงเอง             B. ฉีด steroid
9. ชายอายุ 20 ป MCA หมดสติ ไมสวมหมวกกันน็อค คนพามารพ. มีสติ แผลที่หนา                            C. Consult EYE                        D. consult Plastic Surgery
เลือดไหล เจ็บคอ & อกซาย ถาม management                                                 13. ผูปวยชาย อายุ 50 ป สุขภาพแข็งแรงดี มาตรวจรางกาย ผลปกติดี ผูปวยมีประวัติดื่ม
            A. Hx                                   B. วัด v/s                          สุราและสูบบุหรี่มานาน หลายป ไมไดใชยาใดเปนประจํา ผล investigation พบวามี
            C. ใส soft collar                      D. ใส Foley catheter               Cholesterol 210, LDL 120, HDL 55, TG 150 มีปญหาเรื่องอวัยวะเพศไมแข็งตัวกับภรรยา
            E. พัน EB ที่หัว                                                            จึงลดการสูบบุหรี่ลง มาปรึกษาทาน ทานจะทําอยางไร
A. ใหบันทึก morning erection ทุกเชา   B. สงปรึกษาจิตแพทยตอไป                                    ORTHO-REHAB
C. ใหยาเพิ่มการแข็งตัว                 D. ปรึกษา urological surgery
                                                                       1. ชายชาวพมา 18 ป ปวดเขาซายอยู 2 วันแลวหายเอง ตอมาปวดขอเทาซาย มีไข 38.2 P
E. เปนปกติตามวัย
                                                                       102/min BP ปกติ ใหยา
                                                                                  A. ASA                               B. paracetamol



                                                                                                         6
                                                                                  C. Augmentin                         D. doxycycline




                                                                                                      11
                                                                       2. ortho ชายดื่มสุรา ไมสามารถทํา finger extendได extend wrist ไมได



                                                                                I
                                                                                  A. Radial nerve palsy



                                                                              S
                                                                       39) ผูปวยหญิงอายุ 75 ป ผาตัดเปลี่ยนขอสะโพกมานาน 6 วัน ใหเลือก gait aid ที่เหมาะสม



                                                                            t
                                                                       แกผูปวยรายนี้




                                                  igh
                                                                                  A. Walker                            B. Tripod cane




                                               yr
                                                                                  C. Axillary crutch
                                                                                                          EYE-ENT


                                   o p
                                                                       1. รูป perichonditis




                       C
                                                                                  A. perichonditis
                                                                                  B. Bezoid
                                                                                  C. cellulites
                                                                       2. ชายอายุ 50 ป ปวดใตคอ คอบวม กลืนลําบาก ได antibiotic แลวอาการไมดีขึ้น ควรทํา
                                                                       อะไรตอ
                                                                                  A. tracheostomy + I&D            B. I&D
                                                                                  C. ET tube + I&D
3. ผูปวยชายประสบอุบัติเหตุที่ใบหนาดานขวา ตาบวมปด มีเมือกใสๆติดอยูที่กระจกตา                    C. epidermia keratoconjunctivitis D. bacterial keratoconjunctivitis’
กอนสงไปพบจักษทแพทย ควรทําอะไรกอน                                                     8. ผูปวย acute angle closure glaucoma ควรใหยาอะไร
           A. ปด Eye shield                     B. Topical antibiotics                              A. Timolol
           C. ใหยาลด Intra-ocular pressure                                              9. ผูปวย Acute angle closure glaucoma และมี cataract รวมดวย ควรนึกถึงภาวะใด




                                                                                                                           6
4. เด็กอายุ 3 ป ไข ไอ ปวดหูมากมา 1 วัน รักษาอยางไร                                                A. Phagomorphic glaucoma



                                                                                                                         1
           A. Myringotomy                        B. Oral ATB




                                                                                                  I                     1
           C. Topical ATB




                                                                                                S
5. ผูปวยหญิงอายุ 30 ป ทํางานโรงงานเย็บผา หูขวาไดยินลดลงเรื่อยๆ




                                                                                              t
PE : tympanic membrane intact bilaterally



                                                                      h
Weber’s test lateral to the right




                                                                    ig
Rinne’s test AC<BC at right ear, normal at left ear จงใหการวินิจฉัย




                                                                 yr
           A. Otosclerosis                       B. Noise induced hearing loss




                                               p
           C. Cholesteatoma                      D. Acoustic neuroma




                                             o
6. Corneal abrasion ทํา pressure patch 2 วัน ตอมาตรวจตาพบวามีจุดสีขาวที่กลางตา




                                 C
ขนาด 3 mm ตรวจรางกายอื่นๆปกติ ควรทําอยางไร
           A. Topical ATB                        B. Oral ATB
           C. Topical steroid                    E. ขูด lesion ออกจนหมด
7. หญิง 35 ป เคืองตาขวามา 5 วัน ตรวจตา VA 6/24 , 6/6 ตาขวา – watery discharge ,
mild conjunctival injection with follicles , กระจกตามี subepithelial infiltration เปน
หยอมๆ จงDx
           A. atropic keratoconjunctivitis       B. vernal keratoconjunctivitis
PHYCHI                                                          A. normal grief
                                                                                                    B. adjustment disorder
1. ผูหญิงอายุ 45 ป ระแวงวาจะมีคนที่ตนเคยไปตอวาจะมาทําราย วินิจฉัยอะไร
                                                                                                    C. acute stress disorder
           A. Schizophreniform disorder        B. Delusional disorder
                                                                                                    D. PTSD
2. ผูปวยหญิง ถูกแฟนทิ้งมีอาการ depressed mood มานาน 1 ป ชวง 2 สัปดาหนี้มีอาการรา



                                                                                                                          6
                                                                                                    E. brief psychotic
เริง ชอบซื้อของแจกเพื่อน บอกผูอื่นวาตนมีแฟนเปนนักรอง โทรจิตหานักรองได ควรให



                                                                                                                        1
                                                                                         6. ชาย 50 ป สับสน 1 วัน ภรรยาให disulfuram กินในเหลา ติดเหลา 20 ป ถามสับสนจาก



                                                                                                                       1
ยาใด



                                                                                                  I
                                                                                                    A. alcohol                          B. delirium tremen
           A. Lithium + Diazepam               B. Lithium + Fluoxetine



                                                                                                S
                                                                                                    C. disulfuram                       D. acetaldehyde
           C. Fluoxetine                       D. Haloperidol + Chlorpromazine



                                                                                              t
                                                                                                    E. dopamine
           E. Haloperidol



                                                                    h
                                                                                         7. หญิงอายุ 21 ปเดินผานพระพุทธรูปแลวนึกคําดา คําหยาบตลอด จนรูสึกอึดอัด ใหยา




                                                                  ig
3. ผูปวยเปน DM สูบบุหรี่จัด แพทยแนะนําใหหยุด แตผูปวยไมพรอมหยุด ควรทําอยางไร
                                                                                         อะไร




                                                               yr
           A. บอกขอดีขอเสียของบุหรี่
                                                                                                    A. fluoxetine
           B. ใหโอกาสผูปวยกลับมาปรึกษาเมื่อพรอม



                                              p
                                                                                                    B. lorazepam
           C. ใหกําลังใจในการเลิกบุหรี่



                                            o
                                                                                                    C. promethazhe
4. ชายคิดวาแฟนตัวเองนอกใจคบกับเพื่อนตัวเอง ไดยินเสียงหูแววเปนเสียงเพื่อนจูจี๋กับ



                                C
                                                                                                    D. haloperidol
แฟน เปนมา 1 ป รูสึกวามีคนกําลังอานความคิดของตนและคิดวาเพื่อนเอาไมโครชิปฝงใน
                                                                                                    E. carbamazepine
หัวตัวเอง ไมมีซึมเศราหรืออารมณดีผิดปกติ จงใหการวินิจฉัย
                                                                                         8. หญิงอายุ 40 ป 1 mo มีปญหากับเพื่อนรวมงาน นอนไมหลับ เบื่อหนาย รูสึกผิด , 3 วัน
           A. schizophrenia                    B. schizoaffective
                                                                                         กอน รูสึกเพื่อนกลั่นแกลง คิดถึงความตายตลอด ถาม management
           C. schizophreniform                 D. delusional disorder
                                                                                                    a. admit
           E. S….red(อานไมออก) delusion
                                                                                                    b. fluoxitin
5. ทหารโดนยิงตอนสูกับผูกอการรายรักษาหานดี1 สัปดาห มีฝนรายนอนไมหลับตกใจกับ
                                                                                                    c. fluoxitin + diazepam
เสียงดัง สิ่งแวดลอมเปลี่ยนไป
d. fluoxitin + …                                                                                                   FORENSIC
           e. diazepam + …
                                                                                            1. ผูหญิง(อายุมากมั๊ง)ถูกตี -> Rt femur Fx -> รักษาโดยเหล็กดาม -> death จาก
9. Hx PANIC DISORDER นอกจากใหยาแลวตองทําอะไร
                                                                                            ภาวะแทรกซอน ชันสูตรพบ pneumonia ขอใดคือพฤติการณตายตามกฏหมายอาญา
           A. breathing exercise
                                                                                                       A. Natural death                        B. Homicide



                                                                                                                              6
10. ผูปวยอายุ 50 ป เปน AIDs มา 5 ป , ครั้งนี้เปน PCP มา 2 wks. รักษาอาการดีขึ้นมา 1
                                                                                                       C. Accident                             D. Pneumia



                                                                                                                            1
wk , จากนั้นมีอาการ disoriented to time, place & person , เห็นภูติผี , พูดคนเดียว ถามวา



                                                                                                                           1
                                                                                                       E. Rt Femur Fx



                                                                                                     I
เปนโรคอะไร
                                                                                            2. ผูชวยพยาบาลถูกเข็มตํานิ้ว โดยเข็มที่ตําเปนเข็มเจาะเลือดผูปวยเมาสุรา หลังทําความ



                                                                                                   S
           A. delirium
                                                                                            สะอาดแผลแลว ควรทําอยางไรตอไป



                                                                                                 t
11. ผูปวยชายไทยอายุ 40 ป ดื่มเหลามาตลอด มาอยูโรงพยาบาลผาตัด whipper operation ~
                                                                                                       A. ขอผูปวยสงเลือดตรวจ HIV



                                                                      h
3 วัน สับสนโวยวาย ปนเตียง เห็นสายน้ําเกลือเปนงู




                                                                    ig
                                                                                                       B. รอเจาะตรวจเลือดอีก 6 mth
           A. Delirium




                                                                 yr
                                                                                                       C. ใหคําแนะนําวามีโอกาสติดเชื้อนอยมาก
           B. Delirium trimen
                                                                                                       D. สงเลือดผูชวยพยาบาลตรวจ HIV



                                                p
                                                                                            3. ผูปวยชาย เปน DM, HT, CAD, CKD ชวยเหลือตัวเองพอได ไมอยากอยูกับลูกหลาน จึง



                                              o
12. หญิงอายุมาก สามีเสียชีวิต ลูกรับไปอยูดวย มีอาการนอนไมหลับ ไมกินขาว เกาศีรษะ
                                                                                            อยูกับภรรยา โดยใหภรรยาชวงพยุงเดินและชวยจัดยา วันหนึ่งมี heart failure ทํา CPR



                                 C
           A. Depression
                                                                                            ตอมาไตวาย ตองลางไต 3 ครั้ง/สัปดาห ผูปวยมีอาการซึมเศรา เบื่ออาหาร น้ําหนักลด
13. ผูปวยหญิงสูงอายุ ชวงนี้มีอาการหลงลืมบอย อาการอื่นๆปกติ ตรวจ MMSE ได 21
                                                                                            อยากตาย ไมอยากรักษา อยากขอไมลางไต ควรทําอยางไร
(เต็ม 30)
                                                                                                       A. ยอมผูปวย แตใหเซ็นตหนังสือปฏิเสธการรักษา
           A. Dementia                                   B. Delirium
                                                                                                       B. บอกภรรยาใหรับทราบ และเซ็นตหนังสือยินยอม
           C. Mild cognitive impairment                  D. Depression
                                                                                                       C. ใหภรรยาแสดงความเห็นและรวมตัดสินใจ
                                                                                                       D. ใหลูกหลาน แสดงความเห็นและรวมตัดสินใจ
                                                                                                       E. พูดคุยกับผูปวยและรักษาภาวะซึมเศรา
4. ศพชายไทย มีรอยดําเปนเสนรอบคอ พาดจากคอดานหนาไปยังบริเวณใบหู ถามวา                          E. หญิงเอาอวัยวะเทียมใสรูทวารหนักหญิง
เสียชีวิตจากอะไร (รูป)                                                                 8. ในการสอบคดี ศาลเชื่อวาขณะที่ผูตายตายนั้น ผูตองหาอยูหางจากผูตาย 2m การตรวจ
           A. Hanging                                      B. Electrocution            ชันสูตรบาดแผลอยางไรที่ทําใหศาลยกฟอง
           C. Manual strangulation                         D. ligature strangulation              A. แผลเปนลักษณะ satellite และมีดินปนใตแผล




                                                                                                                        6
           E. Blunt neck injury                                                                   B. บาดแผลกลมขอบเรียบ



                                                                                                                      1
5. พบศพหญิง เสียชีวิตในทานอนหงาย ตาเปด กระจกตาขุน มี fixed rigor, rivor กดไม                  C. บาดแผลมีเขมาดินปนรอบแผล




                                                                                                I                    1
จาง ทองมีสีเขียวคล้ํา ขอใดบงบอกวาผูปวยนาจะเสียชีวิตมามากกวา 24 hr                         D. ลักษณะแผล รูปกลม มีรอยถลอกที่ขอบแผล




                                                                                              S
           A. กระจกตาขุน                        B. Fixed rigor mortis                            E. รูปรี มีเขมา




                                                                                            t
           C. Livor mortis กดไมจาง              D. ทองสีเขียวคล้ํา                   9. ผูปวยชาย โดนทําราย ไมทราบชนิดอาวุธ พบเปนรอยแผล cut wound บริเวณใบหนา



                                                                   h
6. เด็กเปนมะเร็งที่จอประสาทตา หมอใหเอาตาออก แมยินยอมแตพอไมยินยอม ตองทํา         ยาว 6 cm. ลึก 0.5 cm., มี rupture globe ของ Lt. eye ทานจะเขียนในใบชันสูตรบาดแผล




                                                                 ig
อยางไร                                                                                วาอยางไร




                                                              yr
           A. เอาตาออก เพราะอันตรายถึงชีวิต                                                       A. อันตรายบาดเจ็บสาหัส




                                              p
           B. เอาตาออก เพราะแมยินยอม                                                             B. อาจเปนแผลเปนได




                                            o
           C. เอาตาออก และให ผอ รพ เซ็น                                                          C. แผลสามารถมองเห็นไดจากระยะ 5 m




                               C
           D. เอาตาออก และใหปูเซ็นยินยอม                                                        D. แผลเสียโฉมที่หนาแบบติดตัว
           E. ไมเอาตาออก เพราะพอไมยินยอม                                                       E. อาจทําใหตาบอดได
7. นิติเวช ขอใดความผิดฐานกระทําชําเรา                                                 10. ผูปวยหญิงอายุ 15 ป มีประจําเดือนผิดปกติ LMP 14 d PTA ถูกขมขืนมา 5 hr PTA
           A. ชายเอาอวัยวะเทียมใสปากหญิง                                              ตรวจรางกายพบรองรอยวาถูกกระทําชําเรา จะใหการรักษาอยางไร
           B. ชายเอาอวัยวะเทียมใสปากชาย                                                          A. ตรวจ ALP                          B. ตรวจ UPT
           C. หญิงเอาอวัยวะเทียมใสปากหญิง                                                        C. ให Antibiotics                   D. ให levonorgestrel
           D. ชายเอาอวัยวะเทียมถูไถรูทวารหนักหญิง                                                 E. reassure หากประจําเดือนขาด ใหมารพ.

Más contenido relacionado

Destacado

Compre Rama 2010
Compre Rama 2010Compre Rama 2010
Compre Rama 2010vora kun
 
Tutorial national board 2010 Nephrology
Tutorial national board 2010 NephrologyTutorial national board 2010 Nephrology
Tutorial national board 2010 Nephrologyvora kun
 
Nt2009 complete all
Nt2009 complete allNt2009 complete all
Nt2009 complete allvora kun
 
ortho 06 common ortho dis 2 edited 12 mar 10
ortho 06 common ortho dis 2 edited 12 mar 10ortho 06 common ortho dis 2 edited 12 mar 10
ortho 06 common ortho dis 2 edited 12 mar 10vora kun
 
Step3 Tutorial by SWU book2
Step3 Tutorial by SWU book2Step3 Tutorial by SWU book2
Step3 Tutorial by SWU book2vora kun
 
Abnormal pap smear ศิริราช ppt
Abnormal pap smear ศิริราช pptAbnormal pap smear ศิริราช ppt
Abnormal pap smear ศิริราช pptvora kun
 
ortho 02 orthopaedic complication & prevention + orthopaedic trauma (practica...
ortho 02 orthopaedic complication & prevention + orthopaedic trauma (practica...ortho 02 orthopaedic complication & prevention + orthopaedic trauma (practica...
ortho 02 orthopaedic complication & prevention + orthopaedic trauma (practica...vora kun
 
Step3 Tutorial by SWU book1
Step3 Tutorial by SWU book1Step3 Tutorial by SWU book1
Step3 Tutorial by SWU book1vora kun
 
ortho 04 drugs in orthopaedic (principle & common use)
ortho 04 drugs in orthopaedic (principle & common use)ortho 04 drugs in orthopaedic (principle & common use)
ortho 04 drugs in orthopaedic (principle & common use)vora kun
 
ortho 03 principle of closed reduction in fracture and dislocation
ortho 03 principle of closed reduction in fracture and dislocationortho 03 principle of closed reduction in fracture and dislocation
ortho 03 principle of closed reduction in fracture and dislocationvora kun
 

Destacado (10)

Compre Rama 2010
Compre Rama 2010Compre Rama 2010
Compre Rama 2010
 
Tutorial national board 2010 Nephrology
Tutorial national board 2010 NephrologyTutorial national board 2010 Nephrology
Tutorial national board 2010 Nephrology
 
Nt2009 complete all
Nt2009 complete allNt2009 complete all
Nt2009 complete all
 
ortho 06 common ortho dis 2 edited 12 mar 10
ortho 06 common ortho dis 2 edited 12 mar 10ortho 06 common ortho dis 2 edited 12 mar 10
ortho 06 common ortho dis 2 edited 12 mar 10
 
Step3 Tutorial by SWU book2
Step3 Tutorial by SWU book2Step3 Tutorial by SWU book2
Step3 Tutorial by SWU book2
 
Abnormal pap smear ศิริราช ppt
Abnormal pap smear ศิริราช pptAbnormal pap smear ศิริราช ppt
Abnormal pap smear ศิริราช ppt
 
ortho 02 orthopaedic complication & prevention + orthopaedic trauma (practica...
ortho 02 orthopaedic complication & prevention + orthopaedic trauma (practica...ortho 02 orthopaedic complication & prevention + orthopaedic trauma (practica...
ortho 02 orthopaedic complication & prevention + orthopaedic trauma (practica...
 
Step3 Tutorial by SWU book1
Step3 Tutorial by SWU book1Step3 Tutorial by SWU book1
Step3 Tutorial by SWU book1
 
ortho 04 drugs in orthopaedic (principle & common use)
ortho 04 drugs in orthopaedic (principle & common use)ortho 04 drugs in orthopaedic (principle & common use)
ortho 04 drugs in orthopaedic (principle & common use)
 
ortho 03 principle of closed reduction in fracture and dislocation
ortho 03 principle of closed reduction in fracture and dislocationortho 03 principle of closed reduction in fracture and dislocation
ortho 03 principle of closed reduction in fracture and dislocation
 

Más de vora kun

NTstep3round2 9_jan2554
NTstep3round2 9_jan2554NTstep3round2 9_jan2554
NTstep3round2 9_jan2554vora kun
 
Osce examination si116
Osce examination si116Osce examination si116
Osce examination si116vora kun
 
ประชุมวิชาการ ศิริราช 53
ประชุมวิชาการ ศิริราช 53ประชุมวิชาการ ศิริราช 53
ประชุมวิชาการ ศิริราช 53vora kun
 
ประชุมวิชาการ ศิริราช 52
ประชุมวิชาการ ศิริราช 52ประชุมวิชาการ ศิริราช 52
ประชุมวิชาการ ศิริราช 52vora kun
 
Nt2553step3round1 28NOV2553
Nt2553step3round1 28NOV2553Nt2553step3round1 28NOV2553
Nt2553step3round1 28NOV2553vora kun
 
Osce ศรว ครั้งที่สอง 10jan53
Osce ศรว ครั้งที่สอง 10jan53Osce ศรว ครั้งที่สอง 10jan53
Osce ศรว ครั้งที่สอง 10jan53vora kun
 
Osce คณะ si 115
Osce คณะ si 115Osce คณะ si 115
Osce คณะ si 115vora kun
 
Osce ศรว ครั้งแรก dec52
Osce ศรว ครั้งแรก dec52Osce ศรว ครั้งแรก dec52
Osce ศรว ครั้งแรก dec52vora kun
 
Thai Osteoporosis guideline 2553
Thai Osteoporosis guideline 2553Thai Osteoporosis guideline 2553
Thai Osteoporosis guideline 2553vora kun
 
ortho 05 common rheumatic dx rx
ortho 05 common rheumatic dx rxortho 05 common rheumatic dx rx
ortho 05 common rheumatic dx rxvora kun
 
ortho 01 management of open fracture-update by kk 31052010
ortho 01 management of open fracture-update by kk 31052010ortho 01 management of open fracture-update by kk 31052010
ortho 01 management of open fracture-update by kk 31052010vora kun
 
หัตถการที่จำเป็นทางสูติ
หัตถการที่จำเป็นทางสูติหัตถการที่จำเป็นทางสูติ
หัตถการที่จำเป็นทางสูติvora kun
 
SWU CXR interpretation
SWU  CXR interpretationSWU  CXR interpretation
SWU CXR interpretationvora kun
 
Total parenteral nutrition
Total parenteral nutritionTotal parenteral nutrition
Total parenteral nutritionvora kun
 
ศรว 51 By Cmu
ศรว 51 By Cmuศรว 51 By Cmu
ศรว 51 By Cmuvora kun
 
ศรว 51 ANS By Cmu
ศรว 51 ANS By Cmuศรว 51 ANS By Cmu
ศรว 51 ANS By Cmuvora kun
 
Nt2009 Complete Ans
Nt2009 Complete AnsNt2009 Complete Ans
Nt2009 Complete Ansvora kun
 
Mdcu Exam Step 2 2010
Mdcu Exam Step 2 2010Mdcu Exam Step 2 2010
Mdcu Exam Step 2 2010vora kun
 

Más de vora kun (18)

NTstep3round2 9_jan2554
NTstep3round2 9_jan2554NTstep3round2 9_jan2554
NTstep3round2 9_jan2554
 
Osce examination si116
Osce examination si116Osce examination si116
Osce examination si116
 
ประชุมวิชาการ ศิริราช 53
ประชุมวิชาการ ศิริราช 53ประชุมวิชาการ ศิริราช 53
ประชุมวิชาการ ศิริราช 53
 
ประชุมวิชาการ ศิริราช 52
ประชุมวิชาการ ศิริราช 52ประชุมวิชาการ ศิริราช 52
ประชุมวิชาการ ศิริราช 52
 
Nt2553step3round1 28NOV2553
Nt2553step3round1 28NOV2553Nt2553step3round1 28NOV2553
Nt2553step3round1 28NOV2553
 
Osce ศรว ครั้งที่สอง 10jan53
Osce ศรว ครั้งที่สอง 10jan53Osce ศรว ครั้งที่สอง 10jan53
Osce ศรว ครั้งที่สอง 10jan53
 
Osce คณะ si 115
Osce คณะ si 115Osce คณะ si 115
Osce คณะ si 115
 
Osce ศรว ครั้งแรก dec52
Osce ศรว ครั้งแรก dec52Osce ศรว ครั้งแรก dec52
Osce ศรว ครั้งแรก dec52
 
Thai Osteoporosis guideline 2553
Thai Osteoporosis guideline 2553Thai Osteoporosis guideline 2553
Thai Osteoporosis guideline 2553
 
ortho 05 common rheumatic dx rx
ortho 05 common rheumatic dx rxortho 05 common rheumatic dx rx
ortho 05 common rheumatic dx rx
 
ortho 01 management of open fracture-update by kk 31052010
ortho 01 management of open fracture-update by kk 31052010ortho 01 management of open fracture-update by kk 31052010
ortho 01 management of open fracture-update by kk 31052010
 
หัตถการที่จำเป็นทางสูติ
หัตถการที่จำเป็นทางสูติหัตถการที่จำเป็นทางสูติ
หัตถการที่จำเป็นทางสูติ
 
SWU CXR interpretation
SWU  CXR interpretationSWU  CXR interpretation
SWU CXR interpretation
 
Total parenteral nutrition
Total parenteral nutritionTotal parenteral nutrition
Total parenteral nutrition
 
ศรว 51 By Cmu
ศรว 51 By Cmuศรว 51 By Cmu
ศรว 51 By Cmu
 
ศรว 51 ANS By Cmu
ศรว 51 ANS By Cmuศรว 51 ANS By Cmu
ศรว 51 ANS By Cmu
 
Nt2009 Complete Ans
Nt2009 Complete AnsNt2009 Complete Ans
Nt2009 Complete Ans
 
Mdcu Exam Step 2 2010
Mdcu Exam Step 2 2010Mdcu Exam Step 2 2010
Mdcu Exam Step 2 2010
 

NT step2 march 53

  • 1. Medicine National Test 1. ชายไทยอายุ 25 ป มีประวัติชักทั้งตัวมา 2 ครั้ง on phenytoin 300 mg/day หลังจากนั้น ไมมีอาการชักอีก , 3 วันมีไข ชักทั้งตัว , CT : calcified 3 mm. , no edema รอบๆ ถาม 2010 Management 6 A. albendazole B. ใหยาเดิมตอ 11 C. เปลี่ยนเปน Na Valproate D. เปลี่ยนยาเปน ... I E. surgery เอา calcification ออก S 2. ผูปวยอายุ 60 ป สูบบุหรี่ วันละ 20 มวนมานานหลายป มีหอบเหนื่อย ไข เสมหะมาก BY SI116 t มา 1 วัน ผล CXR : พบเปน Perihilar mass (นาจะเปน CA lung) ถามวารักษาอยางไรตอ igh A. chemotherapy B. radiotherapy yr C. surgery D. O2 therapy ฉบับราง 3. ผูปวยไอมีเสมหะ 2-3 ป กินยาเทาไรก็ไมหาย ปากเหม็นมาก p PE : coarse crepitation BLL ถาม diagnosis o A. chronic bronchitis B. COPD C หมายเหตุ ใชสําหรับสอบNational test วันอาทิตยที่10/10/53 C. Bronchiectasis D. lung abscess ขอบคุณเพื่อนๆSI116ทุกคนที่ชวยกันจําขอสอบ 4. หญิง 75 ป เปน Hypertension รักษาดวยยา Hctz(25) 1x1 คุม BP ดีมาตลอด วั้นนี้ลุกจาก ที่นอนไมขึ้น ญาติพามารพ. BP 130/80 , P 90 /min , alert , on wheel chair , proximal m. ทําโดย กอง(010),First(019),นันทวุฒิ(089),การ(104), weakness , DTR & sensory intact ถามวาเกิดจากอะไร บอส(139),Bank(มง)(144),โย(191),Golf(212) A. hyponatremia B. hypokalemia ยังไงก็ขอใหสอบผานกันทุกคนนะ C. hypomagnesemia D. hypercalcemia E. rhabdomyolysis
  • 2. 5. ชาย 55 ป เปน hepatitis C cirrhosis มีไข ตัว/ตาเหลืองมา 3 วัน ทองโตมา 3 wks. A. tetrodotoxin B. Botulinum toxin PE : T 38 C , Jaundice , tense ascites C. Cyanide D. Cabamate Ascitic fluid study : WBC 2,200 , PMN 60% , mononuclear cell 40% , gram stain neg, E. Organophosphate ascites fluid/serum albumin = 0.9/2.5 mg/dl จง management 9. peripheral blood smear พบ plasmodium falciparum(multiple ring) (รูป) ถามยาที่ใชใน 6 A. somatostatin + albumin 50 g/day การรักษา 1 B. metronidazole 500 mg tid A. Artesunate + mefloquine B. Quinine I 1 C. cef-3 100 mg+ somatostatin IV drip C. Chloroquine + Primaquine D. Chloroquine S D. ciprofloxacin 500 mg + telipresin IV q 6 hr E. Mefloquine t E. cef-3 2 g IV OD 10. ผูปวยสูงอายุ มี U/D เปน HT, glaucoma กินยาไมทราบชนิด แลวมารพ.ดวยอาการซึม h 6. ชายอายุ 55 ป สูบบุหรี่มาก barrel-shape chest , prolong expiratory phase จะสง ลง PE : alteration of consciousness ig investigation อะไร Lab : Na 108, K 3.0, Cl 80, HCO3 35 คิดวาอาการเหลานี้นาจะเกิดจากยาตัวใด yr A. pulmonary function test B. CXR A. Acetazolamide B. Spironolactone p C. CT chest C. HCTZ D. Hydralazine o 7. ชายอายุ 20 ป พบนอนหมดสติ(จากไฟไหม) E. Triamzinorone C V/S : P 68 /min , BP 90/70 mmHg , RR 12 /min , no response to deep pain , พบ black 11. ผูปวยชายสูงอายุ เปน ESRD on hemodialysis สัปดาหละ 2 ครั้ง มีอาการใจสั่น หอบ soot at mouth&thumb ถามวาจะตรวจอะไรเพื่อการวินิจฉัย เหนื่อย ตรวจรางกายพบวามี BP สูง, puffy eyelid, pitting edema both legs A. O2 sat B. Venous blood gas Lab : BUN 60, Cr 6, Na 136, K 6.8, Cl 110, HCO3 27 ควรจะทําอะไรเปนอันดับแรก C. Electrolyte A. NaHCO3 B. Hemodialysis 8. หญิง 25 ปมีอาการชารอบปาก หายใจไมสะดวก , อึดอัด หลังจากรับประทานกวยเตี๋ยว C. Calcium gluconate D. Kayexalate ลูกชิ้นปลามา 1 ชม. หลังจากนอนสังเกตอาการหายใจลําบากมากขึ้นเรื่อยๆ จนหยุดหายใจ E. Glucose + insulin ผูปวยไดรับสารพิษอะไร
  • 3. 12. ผูปวยหญิง มีผื่นที่ศีรษะลักษณะปนสีน้ําตาลที่ศีรษะ. รูปวงกลม ขนาด 5 x 3 cm ตรง 16. ผูชายอายุ 60 ป admit ICU ดวย pneumococaal pneumonia with respiratory failure กลางเห็นเปน capillary ไมมีผมขึ้น, มี scale ตรวจรางกายอื่นๆปกติ วินิจฉัยวาอะไร อาการดีขึ้น ตอมาสําลักอาหารมีไขสูง ไอ หอบเหนื่อยมากขึ้น T 36 CXR มี infiltration, A. Taenia capitis B. Discoid lupus erythematosus sputum C/S : gram neg rods with bipolar staining ให ATB อะไร C. psoriasis D. Kerion A. Amikacin B. Augmentin 6 E. Scleroderma C. Ciprofloxacin D. Cef-3 1 13. ชายอายุ 50 ป poor controlled HT วันนี้ปวดหัว ออนเพลีย BP 200/120 ตรวจรางกาย มี E. Imipenam I 1 sign of congestive heart failure, s3 gallop, retinal hemorrhage จงmanage เบื้องตน 17. ผูปวยชายออนแรงซีกขวา เปนมากขึ้นเรื่อยๆมา 7 วัน ตรวจพบวา no stiff neck, S A. propanolol B. nitroprosside IV positive white patch tongue & oral mucosa จงใหการรักษา t C. hydrolazine IV D. Morphine A. Amphotericin B B. Clotrimazole h E. digoxin C. Co-amoxiclav D. TMP + SMZ ig 14. ผูปวยเคยไดรับการ diag วาเปน Rheumatic fever ตองใหpenicillin จนถึงเมื่อไร E. Ceftriaxone yr A. ใหจนอายุ 50 B. ใหเวลาทําฟน 18. ผูปวยชาย จ.หนองคาย มีอาการปวดบวมแดงรอน ที่บริเวณขาขางซาย drain ไดหนอง p C. ใหตลอดชีวิต พบ bacteria เปน Gram negative slender rod ควรใหยาอะไร o 15. หญิงอายุ 18 ป มีไข, ปวดขอเทาวาย และขอมือขวามา 4 วัน A. Cefazidime B. Gentamicin C PE : T 40 c, BP 110/70 mmHg, PR 80/min, RR 20/min, tenderness and swelling at Left C. Amikacin D. Ceftriaxone ankle joint and right writst joint 19. ผูปวยชาย ชอบกินเหลาประจํา ตรวจพบมี right upper lobe pneumonia, Gram stain ทํา arthrocentesis พบ turbid synovial fluid 5 ml, WBC 25000 (N 90%), Gram stain no เปน numerus Gram negative & positive ควรใหการรักษาอยางไร organism seen จงใหการรักษา A. Co-amoxiclav B. Levofloxacin A. NSAIDs oral B. Ceftriaxone V C. Clarithromycin D. Ceftriaxone C. Penicillin G D. Intraarticular steroid injection 20. ผูชาย 25 ป บริจาคเลือดตรวจพบ HBsAg และ Anti HBC neg, LFT ปกติ ทําอยางไร A. ตรวจ HBsAg ซ้ําใน 6 month B. ให lamivudine
  • 4. C. ตรวจ viral load D. ให hep B immunoglobulin 25. Umbilical vesicle with crust ควรใหยาอะไร E. ให HBV vaccine A. Acyclovir 21. ผูหญิงเหนื่อยงายมา 4 เดือน ตรวจรางกายพบ moderately pale, koilonikis, CBC Hct B. Cloxacillin 22%, wbc 3,500 N 65 L 35 Plt 80,000, PBS ดังรูป (มี Hypersegmental neutrophil, Plt ไม 26. ผูปวยหญิงมีอาการเมาเรือ ทานจะใหยาใด 6 มี) จงใหการวินิจฉัย A. Dimenhydrinate B. Ceterizine 1 A. Apastic Anemia B.Myelopathisis anemia 27. ผูปวยหญิงมีผื่นกลมขึ้นทั่วตัว ผมรวงมีขุย บริเวณรอบๆเห็นเปนเสนเลือด I 1 C. MDS D. iron def A. Tinea capitis B. Discoid rash S E. megaloblastic anemia 28. ผูปวยมีประวัติไอปนเลือดเรื้อรัง ตรวจรางกายพบ dilated superficial vein ที่หนาอก t 22. ผูชายอายุ 22 ป asthma พน Steroid, Salbutamol, ปกติ expire peak 400, RR 35 มี ขางขวา (รูป) ใหการวินิจฉัย h asthma attack -> Peak 220 พน salbutamol x II แลวได PEF 320 RR 25 จะทํายังงัยตอ A. SVC syndrome ig A. Admit B. เพิ่มยา steroid 29. ผูปวยอายุ 50 ป เปน DM ได Metformin + …, BMI 34, FBS 130, HbA1C 8.3 จงให yr C. เพิ่มยา salbutamol D. กลับบาน การ management p 23. ผูปวยหลังรับประทานอาหารทะเลแลว BP drop, มี mucosa บวม, lung clear, มีผื่นขึ้น A. เพิ่มขนาดยา metformin B. เพิ่มยา glipizide o ตามตัว ควรใหการรักษาอยางไร C. เพิ่มยา Acarbose C A. IV corticosteroid B. IM adrenaline 30. ผูปวยหญิงอายุ 30 ป moderately pale, mild icteric sclera C. IV CPM D. IV hydrocortisone Hct 25, WBC 5000, Plt 200000, RBC : anisocytosis & Poikilocytosis 2+ with schitocyte 24. ผูปวยเดินทางไปเที่ยวที่วนอุทยาน 5 wk หลังเดินทางกลับได 2 วันมีไขหนาวสั่น ควร 2+ ควรทําการตรวจอะไรเพิ่มเติม ทํา investigation ใดเพิ่มเติม A. Hb typing B. G-6-PD deficiency A. CBC, H/C C. BMA D. Coomb’s test B. Peripheral blood smear for malaria E. osmotic … C. Blood for rickettsial study
  • 5. 31. ชาย 16 ป ปวดเขา ตรวจ Rt. Knee มี swelling & warm, ผล PT ปกติ & prolonged PTT A. vitamin D excess B. primary hyperparathyroid ใหการรักษาเบื้องตนอยางไร C. secondary hyperparathyroid D. tertiary hyperparathyroid A. Cryoprecipitate B. FFP 37. ผูปวยหญิง 60 ป เปน DM, HT ไดรับประทานยา enalapril, ASA, HCTZ มานาน 1 ป C. Cryoprecipitate + platelet 6 เดือนกอนมีปวดศีรษะ ตรวจพบ uric acid ในเลือดสูง ไดรับยา allopurinol และ 6 32. ผูปวยมีอาการถายอุจจาระบอย ตรวจ AFB พบเชื้อดังรูป (Cryptosporidium) ใหการ paracetamol มารับประทาน 1 วินิจฉัย 1 สัปดาหกอน มี MP rash ขึ้น ถามวานาจะเกิดจากยาใดมากที่สุด I 1 A. Cryptosporidium B. Isospora belli A. ASA B. Enalapril S C. Stongyloides stercoralis C. HCTZ D. allopurinol t 34. ผูปวยหญิงมาดวยเรื่องไข เจ็บคอ E. paracetamol h PE : no thyroid enlargement, tenderness at thyroid gland 38. ผูปวยมีอาการ migraine ถามวายาใดที่ใชปองกันอาการ migraine ไดดีที่สุด ig TFT : FT4 & T3 ↑, TSH ↓ A. ASA B. Propanolol yr A. Acute suppurative thyroiditis B. Subacute thyroiditis C. Indolmethacin D. Etgotamine p C. Grave’s disease D. Hashimoto’s thyroiditis 39. ผูชายอายุ 25 ป ถายเหลวหลังรับประทานอาหารมา 6 เดือน ไมมีอาการอื่นๆ นน. ไม o E. toxic goiter ลด ตรวจอะไร C 35. ชาย 20 ป dx. Grave disease รักษาดวยยา PTU 2x3 หลังรักษามา 3 mo. มา F/U FT4 A. Stool exam B. Colonoscopy สูง , TSH ต่ํา , WBC ~ 500 จง management C. Thyroid Function test D. GI study A. ใหยาขนาดเทาเดิม B. ลดขนาดยาลงเปน 1x3 C. เพิ่มยาเปน 3x3 D. เปลี่ยนยาเปน metrimazole 40. ผูปวยชายอายุ 18 ป หมดสติในงานรับนอง มีกลิ่น alcohol, HR60/min, BP 100/60 E. เปลี่ยนการรักษาเปน Radioactive iodine mmHg ควรทําอยางไรอันดับแรก 36. ผูปวยชายอายุ 70 ป กินวิตามินเสริมประจํา มีประวัตินิ่วเมื่อ 10 ปกอน ตรวจพบ A. ตรวจ ketone B. ตรวจ electrolyte calcium 11, PTH ↑ ถามวาเกิดจากอะไร C. 50% glucose IV push D. CT brain
  • 6. 41. ผูปวยชายอายุ 75 ป BP 140/80 mmHg, PR 38 /min, EKG เปน bradycardia ~ AV 46. ทองเสียเรื้อรัง เดิมเคยทองผูก (คนแก) ทองมี saucesage-like mass RLQ , PR มีอึแข็ง block ควรทําอยางไรตอไป +อึเหลวปนกัน ถาม management (v/s stable , ไมมีอาการอื่นๆ) A. Atropine B. Cardioversion A. ORS B. IV fluid C. Defibrillation D. Adenosine C. loperamide D. NSS enema 6 42. ผูปวยชายอายุ 70 ป มีมือสั่นเวลาหยิบชอน ตรวจรางกายมี normal gait จงใหการ 47. ผูปวยชาย ทําอาชีพขับรถสงของ มีอาการคันหู คันตา จาม น้ํามูกไหล ควรรับประทาน 1 วินิจฉัย ยาอะไร I 1 A. Parkinson’s disease B. Essential tremor A. Loratadine B. CPM S C. Cerebellar atrophy D. Normal change 48. ผูปวยมี diarrhea เปนน้ําเหลวมา 2 วัน มี BP ทานั่ง 80/60 mmHg, ทานอน 120/70 t 43. ผูปวยชายอายุ 40 ป มีซีด ออนเพลียมากขึ้นมา ??? เดือน mmHg, poor skin turgor, JVP = 0, urine Sp.gr. 1.030, urine Na 10 mEq/L ใหการวินิจฉัย h PE : pale, jaundice A. Pre-renal failure B. Acute interstitial necrosis ig CBC : Hct ↓ , WBC ↓ , Plt ↓ C. Acute glomerulonephritis D. Acute tubular necrosis yr UA : Sp.gr. 1.015, RBC 5-10 cells/HF 49. ผูปวยชาย เปน DM ตรวจพบมีปนดําๆที่จมูกและปาก ทํา KOH พบ non-septate p จงใหการวินิจฉัย hyphae with branching o A. G6PD deficiency B. Aplastic anemia A. Clotrimazole B. Itraconazole C C. AIHA D. PNH C. Amphotericin B D. Ketoconazole 44. ผูปวยกลืนน้ํายาลางหองน้ําที่เปนดาง ควรใหการ management อยางไร 50. ผูปวย car accident ตอง exploratory laparotomy + splenectomy ระหวางผาตัดได PRC A. Endoscopy B. Observe 1 U, vital sign stable สงขึ้นไป admit บนตึกผูปวย ไดให PRC อีก 1 U ปรากฏวาผูปวยมี C. ประเมิน score ไขหนาวสั่น CVA tenderness, BP ↓, Pulse rate ↑, RR 24 /min ควรทําอยางไร 45. ผูปวยชายอายุประมาณ 55 ป มี DLP ยาใดตอไปนี้ที่มี cost benefit ในการปองกัน CAD A. ให NSS IV B. ให hydrocortisone A. Simvastatin B. Cholestyramine C. ให dexamethasone D. หยุดการใหเลือด E. ให adenosine
  • 7. 51. ผูปวยชาย มาดวยเรื่อง HT BP 160/100 mmHg ออนแรง ตรวจพบมี Na 152, K 2.2 จง Pedriatric ใหการวินิจฉัย 02 CNS A. primary hyperaldosteronism B. pheochromocytoma 1. ผูปวยเด็กอายุ 10 ป มีอาการเหมอ ครั้งละ 10-20 วินาที วันละประมาณ 10 ครั้งใหการ C. Cushing syndrome management 6 A. Valproic acid B. Diazepam 11 C. Phanobarbital D. Phenytoin I 02 RS S 2. เด็กอายุ 10 เดือนมาดวยอาการหอบเหนื่อย ฟง lungs ได bilateral wheezing พน t salbutamol ไป 1 dose อาการไมดีขึ้น DX igh A. croup B. bronchitis yr C. bronchiolitis D. viral pneumonia E. severe asthmatic attack p 3. ผูปวยเด็กชาย อายุ 10 ป มีอาการไขหวัดมา 3 เดือน ไมมีอาการปวดศีรษะ มีอาการไอ o น้ํามูกไหล C vital sign stable HEENT : no posterior nasal drip, pale nasal mucosa with mucoid discharge จะสงตรวจ อะไรตอไป A. CBC B. Film paranasal sinus C. Allergy test D. nasal discharge culture
  • 8. 4. ผูปวยเด็กเปน asthma on Budesonide 800 mg/day, Salbutamol ตอมามีอาการหอบ 8. ped เด็กอายุ 5 ป เปน dengue hemorrhagic fever BP 90/70 P 120 BW 20 kg ใหสารน้ํา เหนื่อย (Acute asthmatic attack) Peak flow 200 (เดิมได 400) ได oxygen + Salbutamol ใด NB ตอมา Peak flow เพิ่มขึ้นเปน 320 จําทําอยางไรตอไป A. 5% D/N/2 rate 100 ml/hr A. ใหกลับบาน B. เพิ่ม dose Budesonide เปน 1200 B. 5% D/N/2 rate 200 ml/hr 6 C. เพิ่ม dose Sulbutamol D. ให oral prednisolone x 5 days C. 5% D/NSS rate 100 ml/hr 1 5. หญิง 15 ป asthma 5 ป มีไขหวัดมา 1 วัน หอบเหนื่อย RR 30 /min , P 110 /min , BP D. 5% D/NSS rate 200 ml/hr I 1 140/90 mmHg , ได salbutamol nebule จะไดอะไรตอ E. 5% D/N/3 rate 100 ml/hr S A. IV theophylline B. subcut. Terbutaline 9. เด็กทองเสียเปนน้ํา 10 ครั้ง/วันมา 2 วัน อําเจียน 2 ครั้ง t C. IV dexa D. beclomethasone nebule BP 90/50 mmHg , CBC N80% , L 20% เกิดจากเชื้ออะไร h 5. ผูปวยเด็กอายุ 10 เดือน มีหอบ ไข ไอ, wheezing both lung พน salbutamol แลวอาการ A. Salmonella B. Vibrio chorela ig ไมดีขึ้น จงใหการวินิจฉัย C. Strongeloids D. Isospora yr A. croup B. Bronchiolitis E. Amoebesis p C. pneumonia 02 Endocrine o 6. เด็กอายุ 1 ป มีไขต่ําๆ ไอเสียงกอง มีน้ํามูกไหล 10. เด็กหญิงอายุ 10 ป ไมมีอาการอะไรนอกจาก nodular goiter 2cm นอกจาก TFT แลวจะ C PE : inspiratory stridor, subcostal & intercostals retraction จงใหการ management ทําอะไร A. Adrenaline IV B. Salbutamol NB A. urine iodine B. FNA C. Dexamethazone D. ET tube C. thyroid scan 02Infectious 11. เด็กอายุ ... ป , ชวงนี้กินมาก อวนมากขึ้น ตรวจพบ abdominal striae ถามวาจะเกิด 7. เด็ก 6 ป ไขสูง 3 วัน ปวดเมื่อยตัว ถายเหลวเล็กนอย ครูประจําชั้นมา รพ ที่ รร มีเด็กมี ภาวะแทรกซอนใด อาการเหมือนกัน 3-4 คน จงใหการวินิจฉัย A. hypoglycemia B. hypokalemia A. Influenza B. DHF C. hypertension
  • 9. 12. เด็กอายุ 14 ปเปนเบาหวาน อาเจียนมาก BP 90/70, P 110/min, PRR 25/min มา รพ ให 16. เด็กอายุ 10 ป เปน Molluscum contaciosum ที่หลัง ขนาด 0.3-0.8 cm ควรใหการรักษา IV rate อยางไร A. 0.9% NSS 600 cc/hr A. extraction และบีบออก B. ทา sodium thiosulphate B. 0.9% NSS 300 cc/hr C. benzyl peroxide D. Tzank’s smear ผูปวยเด็กมีผื่นตุมแดง 6 C. 0.45% NSS + NaHCO3 600 cc/hr 17. กดเจ็บที่บริเวณขาทั้งสองขาง ใหการวินิจฉัย 1 D. 0.45% NSS + NaHCO3 300 cc/hr A. Erythyma nodosum I 1 E. RLS 300 cc/hr 02Nutrition S 13. เด็กอายุ 5 ป ทองอืดและผูกบอย ผูปวยมีลักษณะลิ้นจุกปาก (รูป) ตรวจวามี BW = P50, 18. ผูปวยเด็กหญิง term กินนมมารดาอยางเดียว อายุขณะนี้ 12 wk มาตรวจรางกายพบ Hct t Ht < P3 จงใหการวินิจฉัยเบื้องตน 32%, WBC&Plt normal, reticulocyte count 2% จะทําอะไรตอไป h A. Achondroplasia B. Thalassemia A. Fe supplement B. Folate supplement ig C. Glycogen storage disease D. Hypothyroidism C. Vitamin E D. Reassure yr E. Hypoparathyroidism 02Neonate p 02 SKIN 19. เด็กแรกเกิดน้ําหนักนอย birth weight 1400 g, jaundice at day2, Apgar score 6,7 ได on o 14. ผูปวยเด็กกอนนอนอาการปกติดี หลังตื่นมามีตุมแดงที่ขาและลําตัว จงใหการวินิจฉัย phototherapy อะไรเปนปจจัยที่ทําใหตองตรวจหูในอนาคต C A. contact dermatitis B. Herpes zoster A. low birth wight B. jaundice 15. dermato เด็กชาย 10 ป มีตุมสีขาวมา 2 เดือน C. on Phototherapy D. Apgar score ต่ํา PE : multiple discrete umbillicated papule ผื่นเปนมากขึ้นตามรอยเกา 20. เด็กแรกคลอดไมหายใจ ตัวเขียว ชวยหายใจ บีบ bag นาน 5 นาที เขียวมากขึ้น หนาอก A. Extraction B. Topical Na thiosulfate โปง ทองแฟบ HR 60/min เสียงหัวใจฟงชัดดานขวา ควรทําอยางไร C. Topical Benzyl benzoate D. Itraconazole A. Adrenaline IV B. เจาะ Lt. pleura E. Acyclovir B. ET tube
  • 10. 21. ผูปวยเด็กแรกเกิดหนัก 4200 g มารดาเปน DM พบวามีใบหนาแดงคล้ํา Hct = 70% , OB-GYN blood sugar = 45 ควรทําอะไรตอไป 03GYN A. Partial blood transfusion B. Blood testing 1. ผูปวยหญิง G1 ขาดประจําเดือนมา 2 เดือน มีเลือดออกกระปริดกระปรอยทางชองคลอด C. NSS iv D. 10% D/W ปวดทองนอย ตรวจรางกายมี vital sign stable, BP 80/50 mmHg 6 E. early feeding PV : adnexal mass, bulging cul-de-sac, tender at cervical excitation 1 22. เด็กหลังคลอด 10 hr พบมี Jaundice MB = 10 , Hct = 48% 1 ใหการวินิจฉัย I a. on photo b. exchange A. Rupture ectopic pregnancy S 02Emergency 2. หญิง GA 30 wks , bp สูง , ลูกไมมี FHR , CBC : MAHA , จะตองเจาะอะไร t 23. ผูปวยเด็กขณะเลนลูกแกว มีไอ สําลัก แตยังสามารถเลนไดตามปกติ ตรวจมี SpO2 A. LFT B. Lupus andicoag h 94% ควรทําอยางไร ig C. D-dimer A. ใส ET tube B. tracheostomy yr 3. ใหรูป candida albicans มีประวัติตกขาว คัน ใหการรักษา C. direct laryngoscopy + bronchoscopy A. amoxicillin B. metronidazole p 02 Delvolopement C. Norfloxacin D. vaginal tablet metronidazole o 24. ผูปวยเด็ก มีลักษณะซุกซนมาก อยูไมนิ่ง เรียกไมหัน ชอบเลนคนเดียว ควรสงตรวจ E. vaginal tablet clotrimazole C อะไร 4. ผูปวยหญิงอายุ 30 ป ยังไมเคยตั้งครรภ ไมมีอาการใดผิดปกติ มาตรวจรางกาย ทํา A. Denver test ultrasound พบวามี uterus size = 11 cm, มี frbroid mass 3x2 cm ที่ anterior wall และ 2x2 02KUB cm ที่ posterior wall ควรทําอยางไร 25. เด็กชายอายุ 1 ป มีไขมา 1 วัน ไดวินิจฉัยเปน UTI ปสสาวะพบ numerous WBC, A. F/U 3 months B. GnRH agonist Culture E. coli ให antibiotic นาน 2 สัปดาห อาการดีขึ้น ควรทําการสืบคนใดเพิ่มเติม C. Uterine artery embolization D. Myomectomy A. VCUG B. IVP E. Hysterectomy C. urine culture q 1 mth
  • 11. 5. ผูปวยหญิง ตรวจรางกายพบมีลักษณะบวมแดงที่ vulva สงสัย masupial cyst ควรทํา E. induction of labor อะไรตอ (รูป) 9. ผูปวยหญิงตั้งครรภ เคยไดรับวินิจฉัยเปน placenta previa เมื่อ GA 32 wk ไดรับฉีด A. Reassure B. cruciate incision & drainage steroid IM ไป เมื่อ GA 34 wk ครั้งนี้มีเลือดออกชุมผาอนามัย 2 ผืน เมื่อ 4 hr กอนมารพ. C. Enucleation D. Excision แตไมมีอาการเจ็บครรภ ไมมีมดลูกบีบตัว บานอยูหางจากรพ.เดินทางประมาณ 1 hr ผูปวย 6 E. Vulvectomy ตองการคลอดตามกําหนดอีก 1 สัปดาห ควรทําอยางไรตอไป 1 6. ผูปวยหญิง มีปญหามีบุตรยากมา 4 ป มีประวัติถูก rape มา 5 วันกอนและตอมาตั้งครรภ A. Admit + observe I 1 ควรทําอยางไร B. Discharge แลวนัดมาผาตัดคลอดตามนัด S A. abortion B. ใหผูปวยปรึกษาสามี C. ผาตัดคลอดทันที t 7. หญิงอายุ 17 ป ไข ปวดทอง 2 วัน 10. ผูหญิง 40 ป แตงงานมา 10 ป GA 22 wks ตรวจน้ําคร่ําได 47,XY trisomy 21 ควรให h PE : T39.8 c, BP 110/70 mmHg, RR 20/min, P 90/min คําแนะนําแกมารดาอยางไร ig Abd : rebound tenderness at lower abdomen A. ไดลูกเปนผูชาย มีโอกาสมีความผิดปกติ yr PV : mucopurulent discharge, cervical motion tenderness, normal uterine size, Adnexa B. ใหขอมูลบิดาและมารดา แลวใหรวมกันตัดสินใจ p tenderness both sides C. สง ultrasound เพิ่มเติม o ใหการวินิจฉัย D. MRI เพิ่มเติม C A. Acute PID B. Acute appendicitis E. แนะนําใหยุติการตั้งครรภ C. Ruptured endometriotic cyst D. acute enterocolitis 11. ผูปวยหญิง GA 10wk, HBsAg positive ใหวางแผนการรักษา 03OBS A. avoid breast feeding B. ให vaccine 8. ผูปวยหญิงตั้งครรภ GA 39 wk มาดวยเรื่องวาลูกไมดิ้นมา 24 hr ตรวจพบ FHR 140/min C. ให HBIg ควรทําอยางไร 12. ผูปวยหญิงอายุ 30 ป G1 GA 34 wk ปวดศีรษะ ตามัว BP 160/100 mmHg, urine A. reassure & F/U วันรุงขึ้น B. Cesar. protein 3+ ตรวจ fundoscopic exam มี papilledema ควรทําอะไรเปนลําดับตอไป C. Oxytocin stress test D. BPP A. IM dexamethasone B. Induction of labor
  • 12. C. Magnesium IV Surgery 13.. หญิง 40 ป ตั้งครรภ 5 ปวดเมนสมา 2 เดือนเศษๆ มีเลือดออกกระปบกระปอยทาง 1. ตอขวด ICD 3 ขวด แลวพบมีลมปุดที่ขวดที่ 2 ตลอดเวลา พอ clamp สายแรกแลวหาย ชองคลอดมา 1 wk P.E. fundus 1/3 เหนือ pubic sym, BP 140/92 mmHg, P 102/min, HR ถามวา รั่วตรงไหน 95/min ตรวจพบ bilat cystic mass at adnexa 3 cm 2. ผูปวยชาย 50 ป มีอาการปวดทองบริเวณลิ้นปมา 24 ชม. จากนั้นปวด RLQ กดเจ็บ 6 A. molar pregnancy B. ectopic pregnancy ถามวา investigation ใดแมนยําที่สุด 1 C. threaten abortion 1 A. Plain abdomen B. GI follow through I C. Barium D. CT S E. U/S t 3. ผูปวยหญิงอายุ 35 ป มีอาการปวดบริเวณเตานมขวา igh PE : mild swelling and tenderness LUO quadrant, no palpable mass, no axillary mass yr ควรตรวจขึ้นตนดวยอะไร A. CBC B. CA 19-9 p C. mammogram D. FNA o 4. ผูปวยชาย อายุ 70 ป ตรวจพบมีตอมลูกหมากโต PR : rubbery consistency ไมมีอาการ C ผิดปกติใด PSA = 3 จงใหการ management A. Follow up B. alpha-blocker C. TURP C. 5-alpha reductase inhibitor 5. ผูปวยชายประสบอุบัติเหตุ on ET tube นาน 2 wk ไดรับอาหารทาง NG tube ได total cal. 1800 kcal/day, TC 60 ควรไหสารอาหารแบบใด A. Kcal 2000 kcal/day, TC 60 B. Kcal 2400 kcal/day, TC 90
  • 13. C. Kcal 2800 kcal/day, TC 120 10. ชายอายุ 65 ป ปสสาวะกระปริดกระปรอย เบงนานกวาจะหมดมา 3 เดือน 1วันกอนมี D. เพิ่มไข 2 ฟอง/วัน ไข น้ํามูก กินยาเอง วันนี้ปสสาวะไมออก E. กินไขทุกมื้อ มื้อละ 1 ฟอง PE : full bladder, prostate enlargement, rubbery consistency ควรใหการรักษาอยางไร 6. ผูปวยมีแผลที่ริมฝปากดานในขนาด 1 cm มานาน 7 วัน ตรวจพบ LN ที่คางดานซายโต A. ใสสายสวนปสสาวะ 3-5 วัน 6 เสนผานศูนยกลางขนาด 1 cm with tenderness ควรทําอะไรตอ B. ใสสายสวนปสสาวะ 3-5 วัน + ใหยารักษา BPH 1 A. observe B. LN biopsy C. ใสสายสวนปสสาวะนาน 3-5 วัน + TURP I 1 C. incisional biopsy of oral ulcer D. Excisional biopsy of oral ulcer D. Suprapubic cystostomy S E. FNA LN E. Cystoscopy หาสาเหตุของการปสสาวะไมออก t 7. ชายอายุ 50 ป, เคยปวดเอว,มีไข,ปสสาวะแสบ 2 ครั้ง/ป เคยปวดเอวราวลงขาขวาเมื่อ 2 11. ผูปวยหญิง ผาตัด thyroid mass Lt lobe เสนผาศูนยกลางขนาด 3 cm หลังผาตัดมีเสียง h ปกอน 2 ครั้ง , U/A : RBC 5-10 , WBC 0-1 , BUN 15 , Cr 0.5 จะสง Investigation อะไร แหบ กลืนลําบาก เนื่องจากอะไร ig A. urine Ca B. plane KUB A. laryngeal edema B. injury to superior laryngeal n. yr C. U/S KUB D. cystoscope C. arythenoid subluxation D. vocal cord paralysis p E. IVP 12. เด็กอายุ 2 ป มีกอนสีแดงที่หนังตาซาย ตอนแรกมีลกษณะคลายตุมยุงกัด ตอมากอนโต ั o 8. ผูปวย trauma มีไอเปนเลือด , คลํา crepitation ไดที่คอ มากขึ้นเรื่อยๆลักษณะเปนกอนสีแดงคลายสตรอเบอรี่ มีหนังตาตกลงมาครึ่งหนึ่ง จะทํา C A. ABG B. intubation อยางไร C. tracheostomy A. observe ใหกอนยุบลงเอง B. ฉีด steroid 9. ชายอายุ 20 ป MCA หมดสติ ไมสวมหมวกกันน็อค คนพามารพ. มีสติ แผลที่หนา C. Consult EYE D. consult Plastic Surgery เลือดไหล เจ็บคอ & อกซาย ถาม management 13. ผูปวยชาย อายุ 50 ป สุขภาพแข็งแรงดี มาตรวจรางกาย ผลปกติดี ผูปวยมีประวัติดื่ม A. Hx B. วัด v/s สุราและสูบบุหรี่มานาน หลายป ไมไดใชยาใดเปนประจํา ผล investigation พบวามี C. ใส soft collar D. ใส Foley catheter Cholesterol 210, LDL 120, HDL 55, TG 150 มีปญหาเรื่องอวัยวะเพศไมแข็งตัวกับภรรยา E. พัน EB ที่หัว จึงลดการสูบบุหรี่ลง มาปรึกษาทาน ทานจะทําอยางไร
  • 14. A. ใหบันทึก morning erection ทุกเชา B. สงปรึกษาจิตแพทยตอไป ORTHO-REHAB C. ใหยาเพิ่มการแข็งตัว D. ปรึกษา urological surgery 1. ชายชาวพมา 18 ป ปวดเขาซายอยู 2 วันแลวหายเอง ตอมาปวดขอเทาซาย มีไข 38.2 P E. เปนปกติตามวัย 102/min BP ปกติ ใหยา A. ASA B. paracetamol 6 C. Augmentin D. doxycycline 11 2. ortho ชายดื่มสุรา ไมสามารถทํา finger extendได extend wrist ไมได I A. Radial nerve palsy S 39) ผูปวยหญิงอายุ 75 ป ผาตัดเปลี่ยนขอสะโพกมานาน 6 วัน ใหเลือก gait aid ที่เหมาะสม t แกผูปวยรายนี้ igh A. Walker B. Tripod cane yr C. Axillary crutch EYE-ENT o p 1. รูป perichonditis C A. perichonditis B. Bezoid C. cellulites 2. ชายอายุ 50 ป ปวดใตคอ คอบวม กลืนลําบาก ได antibiotic แลวอาการไมดีขึ้น ควรทํา อะไรตอ A. tracheostomy + I&D B. I&D C. ET tube + I&D
  • 15. 3. ผูปวยชายประสบอุบัติเหตุที่ใบหนาดานขวา ตาบวมปด มีเมือกใสๆติดอยูที่กระจกตา C. epidermia keratoconjunctivitis D. bacterial keratoconjunctivitis’ กอนสงไปพบจักษทแพทย ควรทําอะไรกอน 8. ผูปวย acute angle closure glaucoma ควรใหยาอะไร A. ปด Eye shield B. Topical antibiotics A. Timolol C. ใหยาลด Intra-ocular pressure 9. ผูปวย Acute angle closure glaucoma และมี cataract รวมดวย ควรนึกถึงภาวะใด 6 4. เด็กอายุ 3 ป ไข ไอ ปวดหูมากมา 1 วัน รักษาอยางไร A. Phagomorphic glaucoma 1 A. Myringotomy B. Oral ATB I 1 C. Topical ATB S 5. ผูปวยหญิงอายุ 30 ป ทํางานโรงงานเย็บผา หูขวาไดยินลดลงเรื่อยๆ t PE : tympanic membrane intact bilaterally h Weber’s test lateral to the right ig Rinne’s test AC<BC at right ear, normal at left ear จงใหการวินิจฉัย yr A. Otosclerosis B. Noise induced hearing loss p C. Cholesteatoma D. Acoustic neuroma o 6. Corneal abrasion ทํา pressure patch 2 วัน ตอมาตรวจตาพบวามีจุดสีขาวที่กลางตา C ขนาด 3 mm ตรวจรางกายอื่นๆปกติ ควรทําอยางไร A. Topical ATB B. Oral ATB C. Topical steroid E. ขูด lesion ออกจนหมด 7. หญิง 35 ป เคืองตาขวามา 5 วัน ตรวจตา VA 6/24 , 6/6 ตาขวา – watery discharge , mild conjunctival injection with follicles , กระจกตามี subepithelial infiltration เปน หยอมๆ จงDx A. atropic keratoconjunctivitis B. vernal keratoconjunctivitis
  • 16. PHYCHI A. normal grief B. adjustment disorder 1. ผูหญิงอายุ 45 ป ระแวงวาจะมีคนที่ตนเคยไปตอวาจะมาทําราย วินิจฉัยอะไร C. acute stress disorder A. Schizophreniform disorder B. Delusional disorder D. PTSD 2. ผูปวยหญิง ถูกแฟนทิ้งมีอาการ depressed mood มานาน 1 ป ชวง 2 สัปดาหนี้มีอาการรา 6 E. brief psychotic เริง ชอบซื้อของแจกเพื่อน บอกผูอื่นวาตนมีแฟนเปนนักรอง โทรจิตหานักรองได ควรให 1 6. ชาย 50 ป สับสน 1 วัน ภรรยาให disulfuram กินในเหลา ติดเหลา 20 ป ถามสับสนจาก 1 ยาใด I A. alcohol B. delirium tremen A. Lithium + Diazepam B. Lithium + Fluoxetine S C. disulfuram D. acetaldehyde C. Fluoxetine D. Haloperidol + Chlorpromazine t E. dopamine E. Haloperidol h 7. หญิงอายุ 21 ปเดินผานพระพุทธรูปแลวนึกคําดา คําหยาบตลอด จนรูสึกอึดอัด ใหยา ig 3. ผูปวยเปน DM สูบบุหรี่จัด แพทยแนะนําใหหยุด แตผูปวยไมพรอมหยุด ควรทําอยางไร อะไร yr A. บอกขอดีขอเสียของบุหรี่ A. fluoxetine B. ใหโอกาสผูปวยกลับมาปรึกษาเมื่อพรอม p B. lorazepam C. ใหกําลังใจในการเลิกบุหรี่ o C. promethazhe 4. ชายคิดวาแฟนตัวเองนอกใจคบกับเพื่อนตัวเอง ไดยินเสียงหูแววเปนเสียงเพื่อนจูจี๋กับ C D. haloperidol แฟน เปนมา 1 ป รูสึกวามีคนกําลังอานความคิดของตนและคิดวาเพื่อนเอาไมโครชิปฝงใน E. carbamazepine หัวตัวเอง ไมมีซึมเศราหรืออารมณดีผิดปกติ จงใหการวินิจฉัย 8. หญิงอายุ 40 ป 1 mo มีปญหากับเพื่อนรวมงาน นอนไมหลับ เบื่อหนาย รูสึกผิด , 3 วัน A. schizophrenia B. schizoaffective กอน รูสึกเพื่อนกลั่นแกลง คิดถึงความตายตลอด ถาม management C. schizophreniform D. delusional disorder a. admit E. S….red(อานไมออก) delusion b. fluoxitin 5. ทหารโดนยิงตอนสูกับผูกอการรายรักษาหานดี1 สัปดาห มีฝนรายนอนไมหลับตกใจกับ c. fluoxitin + diazepam เสียงดัง สิ่งแวดลอมเปลี่ยนไป
  • 17. d. fluoxitin + … FORENSIC e. diazepam + … 1. ผูหญิง(อายุมากมั๊ง)ถูกตี -> Rt femur Fx -> รักษาโดยเหล็กดาม -> death จาก 9. Hx PANIC DISORDER นอกจากใหยาแลวตองทําอะไร ภาวะแทรกซอน ชันสูตรพบ pneumonia ขอใดคือพฤติการณตายตามกฏหมายอาญา A. breathing exercise A. Natural death B. Homicide 6 10. ผูปวยอายุ 50 ป เปน AIDs มา 5 ป , ครั้งนี้เปน PCP มา 2 wks. รักษาอาการดีขึ้นมา 1 C. Accident D. Pneumia 1 wk , จากนั้นมีอาการ disoriented to time, place & person , เห็นภูติผี , พูดคนเดียว ถามวา 1 E. Rt Femur Fx I เปนโรคอะไร 2. ผูชวยพยาบาลถูกเข็มตํานิ้ว โดยเข็มที่ตําเปนเข็มเจาะเลือดผูปวยเมาสุรา หลังทําความ S A. delirium สะอาดแผลแลว ควรทําอยางไรตอไป t 11. ผูปวยชายไทยอายุ 40 ป ดื่มเหลามาตลอด มาอยูโรงพยาบาลผาตัด whipper operation ~ A. ขอผูปวยสงเลือดตรวจ HIV h 3 วัน สับสนโวยวาย ปนเตียง เห็นสายน้ําเกลือเปนงู ig B. รอเจาะตรวจเลือดอีก 6 mth A. Delirium yr C. ใหคําแนะนําวามีโอกาสติดเชื้อนอยมาก B. Delirium trimen D. สงเลือดผูชวยพยาบาลตรวจ HIV p 3. ผูปวยชาย เปน DM, HT, CAD, CKD ชวยเหลือตัวเองพอได ไมอยากอยูกับลูกหลาน จึง o 12. หญิงอายุมาก สามีเสียชีวิต ลูกรับไปอยูดวย มีอาการนอนไมหลับ ไมกินขาว เกาศีรษะ อยูกับภรรยา โดยใหภรรยาชวงพยุงเดินและชวยจัดยา วันหนึ่งมี heart failure ทํา CPR C A. Depression ตอมาไตวาย ตองลางไต 3 ครั้ง/สัปดาห ผูปวยมีอาการซึมเศรา เบื่ออาหาร น้ําหนักลด 13. ผูปวยหญิงสูงอายุ ชวงนี้มีอาการหลงลืมบอย อาการอื่นๆปกติ ตรวจ MMSE ได 21 อยากตาย ไมอยากรักษา อยากขอไมลางไต ควรทําอยางไร (เต็ม 30) A. ยอมผูปวย แตใหเซ็นตหนังสือปฏิเสธการรักษา A. Dementia B. Delirium B. บอกภรรยาใหรับทราบ และเซ็นตหนังสือยินยอม C. Mild cognitive impairment D. Depression C. ใหภรรยาแสดงความเห็นและรวมตัดสินใจ D. ใหลูกหลาน แสดงความเห็นและรวมตัดสินใจ E. พูดคุยกับผูปวยและรักษาภาวะซึมเศรา
  • 18. 4. ศพชายไทย มีรอยดําเปนเสนรอบคอ พาดจากคอดานหนาไปยังบริเวณใบหู ถามวา E. หญิงเอาอวัยวะเทียมใสรูทวารหนักหญิง เสียชีวิตจากอะไร (รูป) 8. ในการสอบคดี ศาลเชื่อวาขณะที่ผูตายตายนั้น ผูตองหาอยูหางจากผูตาย 2m การตรวจ A. Hanging B. Electrocution ชันสูตรบาดแผลอยางไรที่ทําใหศาลยกฟอง C. Manual strangulation D. ligature strangulation A. แผลเปนลักษณะ satellite และมีดินปนใตแผล 6 E. Blunt neck injury B. บาดแผลกลมขอบเรียบ 1 5. พบศพหญิง เสียชีวิตในทานอนหงาย ตาเปด กระจกตาขุน มี fixed rigor, rivor กดไม C. บาดแผลมีเขมาดินปนรอบแผล I 1 จาง ทองมีสีเขียวคล้ํา ขอใดบงบอกวาผูปวยนาจะเสียชีวิตมามากกวา 24 hr D. ลักษณะแผล รูปกลม มีรอยถลอกที่ขอบแผล S A. กระจกตาขุน B. Fixed rigor mortis E. รูปรี มีเขมา t C. Livor mortis กดไมจาง D. ทองสีเขียวคล้ํา 9. ผูปวยชาย โดนทําราย ไมทราบชนิดอาวุธ พบเปนรอยแผล cut wound บริเวณใบหนา h 6. เด็กเปนมะเร็งที่จอประสาทตา หมอใหเอาตาออก แมยินยอมแตพอไมยินยอม ตองทํา ยาว 6 cm. ลึก 0.5 cm., มี rupture globe ของ Lt. eye ทานจะเขียนในใบชันสูตรบาดแผล ig อยางไร วาอยางไร yr A. เอาตาออก เพราะอันตรายถึงชีวิต A. อันตรายบาดเจ็บสาหัส p B. เอาตาออก เพราะแมยินยอม B. อาจเปนแผลเปนได o C. เอาตาออก และให ผอ รพ เซ็น C. แผลสามารถมองเห็นไดจากระยะ 5 m C D. เอาตาออก และใหปูเซ็นยินยอม D. แผลเสียโฉมที่หนาแบบติดตัว E. ไมเอาตาออก เพราะพอไมยินยอม E. อาจทําใหตาบอดได 7. นิติเวช ขอใดความผิดฐานกระทําชําเรา 10. ผูปวยหญิงอายุ 15 ป มีประจําเดือนผิดปกติ LMP 14 d PTA ถูกขมขืนมา 5 hr PTA A. ชายเอาอวัยวะเทียมใสปากหญิง ตรวจรางกายพบรองรอยวาถูกกระทําชําเรา จะใหการรักษาอยางไร B. ชายเอาอวัยวะเทียมใสปากชาย A. ตรวจ ALP B. ตรวจ UPT C. หญิงเอาอวัยวะเทียมใสปากหญิง C. ให Antibiotics D. ให levonorgestrel D. ชายเอาอวัยวะเทียมถูไถรูทวารหนักหญิง E. reassure หากประจําเดือนขาด ใหมารพ.